You are on page 1of 75

1.

A 53-year old man books a


consultation with a GP. His main a. Premature rupture of
complaint is a pain in the back. He membranes
is worried about is work as he b. Abruption placental
thinks it would be difficult for him c. Placenta previa
to work as a carpenter. d. High rate of infant
respiratory distress
Which of the following is the best syndrome
way of starting the consultation in e. Less likely to die of sudden
Australian general practice? infant death syndrome

a. Tell me about the pain Answer detail:


b. Where is the pain? The correct answer is e.
c. Is the pain severe? Cigarette smoking is a big health
d. The pain is severe hazard during pregnancy.
e. You want to know the cause
of the pain? Maternal complications of smoking
during pregnancy include:
Answer detail:
The correct answer is a. -Premature rupture of membrane.
-Placental abruption.
The open-ended question is -Placenta previa.
essential in initiating the -High abortion rate.
interview.
Fetal complications due to
A question such as ‘tell me about maternal smoking during
the pain’ says to the patient ‘I’m pregnancy include high rate of
interested in anything you may infant respiratory distress
feel is important enough for you syndrome, high rate of sudden
to tell me’. infant death syndrome and small
for gestational age.
The open-ended question gives
the patient an opportunity to take 3. A 33-year old woman is visiting
temporary control of the you in your primary care office as a
consultation and to outline new patient. She is healthy
problems and concerns. otherwise. Her mother was
diagnosed with colorectal cancer at
Option b) is an example of ‘Direct the age of 50.
Question’. These questions are
useful in later part of the When should this patient be
consultation. screened for colorectal cancer?
Option c) is an example of closed-
ended question. a. At the age of 40
Option d) is an example of a b. At the age of 50
leading question. c. At the age of 60
Option e) is an example of a d. At the age 30
reflective question. e. There is no proven benefit
for colorectal cancer
2. Cigarette smoking is associated screening
with several hazards if mother
smokes during pregnancy. Which Answer detail:
of the following is not a The correct answer is a
complication related to smoking?
Screening for bowel cancer should mild exertion and is very compliant
begin approximately 10 years with his medication. He has come
before the age of diagnosis of to your clinic for advice regarding
colorectal cancer in a first-degree driving advice.
relatives like parent or sibling.
Which of the following is correct
Given that this patient’s mother according to medical fitness
was diagnosed at the age of 50, requirements for driving license in
this patient should have screening Australia?
at age 40.
a. He cannot have commercial
The natural history of a colon driving license
polyp to develop into cancer is b. He is eligible for commercial
thought to be 10 years. driving license
c. He is not eligible for private
Colorectal cancer screening has license
proven mortality benefit and it is d. He is unfit for driving for 6
wrong to say that it has no proven months
benefit. e. He should be reviewed every
week before he goes out for
4. Which of the following vaccine is driving
recommended to use during
pregnancy? Answer detail:
The correct answer is a.
a. MMR
b. Varicella vaccine Person with stable angina that is
c. Monovalent rubella vaccine well managed by medication and
d. Yellow fever does not cause any symptoms on
e. Influenza vaccine mild exertion can drive without
(private) license restriction and
Answer detail: notification to the driver licensing
The correct answer is e. authority yet subject to periodic
monitoring.
All of these (MMR, varicella
vaccine, monovalent rubella However, he cannot have a
vaccine and yellow fever) are live commercial driving license while
attenuated vaccines and are he has a diagnosis of angina and
contraindicated in a pregnant requires medication to help with
woman. his symptoms. This patient
requires regular medication and
In general, live vaccine should not thus is not eligible to hold a
be administered during driving license for a commercial
pregnancy, and women should be vehicle.
advised not to become pregnant 6. A 48-year old man comes to your
within 4 weeks of receiving a live clinic to discuss about his risk of
vaccine. getting bowel cancer. He is
asymptomatic otherwise and has
Influenza vaccination is safe to no personal history of bowel
use in a pregnant woman. cancer, colorectal adenomas or
ulcerative colitis.
5. A 45-year old man presented to
your clinic with a history of angina His mother died at the age of 80
pectoris. He develops chest pain on with urosepsis and his father died
of myocardial infraction at the age 7. You are working as a GP and doing
of 59. Aboriginal and Torres Strait
Islander Adult Health Checks.
However his second cousin was
diagnosed of bowel cancer at the Which of the following is true
age of 56. regarding screening for trachoma
in Australia?
What is most appropriate advice to
this patient? a. In areas where trachoma is
endemic, Aboriginal and
a. FOBT every year starting at Torres Strait Islander people
the age of 50 aged 40-54 years should be
b. FOBT every two years screened every two years
starting at the age of 50 b. In areas where trachoma is
c. Urgent colonoscopy referral endemic, Aboriginal and
d. Sigmoidoscopy Torres Strait Islander people
e. Colonoscopy now and then those aged above 55 years
every five year should be screened every
two years
Answer detail: c. In areas where trachoma is
The correct answer is b. endemic, Aboriginal and
Torres Strait Islander people
If a patient is: aged 30-40 years should be
-asymptomatic screened every year
-no personal history of bowel d. No screening for trachoma
cancer, colorectal adenoma or needed in aboriginal
ulcerative colitis population
-no confirmed family history of e. Only ophthalmologist can do
colorectal cancer in first-degree trachoma screening as this is
relative or second-degree relative eye disease
with colorectal cancer
Diagnosed at the age of 55 over. Answer detail:
Such person should have FOBT The correct answer is a.
every 2 years.
So this patient qualifies this In areas where trachoma or
criteria and should have foecal trichiasis is endemic, Aboriginal
occult blood test every 2 years at and Torres Strait Islander people
the age of 50. aged 40 to 54 years should be
screened every 2 years and those
Urgent colonoscopy referral is not aged above 55 years should be
required at this stage. screened annually for trichiasis as
part of an adult health check.
Sigmoidoscopy is usually
combined with double contrast Then identified patients with
barium enema to enhance the trichiasis should be referred to an
usefulness in bowel cancer eye specialist.
screening. However this is not the
case here. 8. A 45-year old male presented to
Colonoscopy every five years is your office for his 3-yearly fasting
needed in patients who are at high blood sugar check which is normal.
risk of getting colorectal cancer. He has blood pressure 110/80,
heart rate 89/min regular, oxygen
saturation is 98% on room air and
respiratory rate 14/min. he is non- so loud that they sleep in the
smoker and non-alcoholic. What separate room. He feels sleep all
would your advice him? day and reports falling asleep while
waiting at traffic signals.
a. Fasting blood sugar every 10
years On examination, his blood pressure
b. Serum cholesterol is 140/90mmHg. His BMI is 46.
estimation every 5 years
c. Check serum B12 and red You advise him to no to drive the
cell folates truck until a sleep physician
d. Check vitamin D level completes the further assessment.
e. Check blood pressure every He is declining to do that as he
5 years does not want to lose his job.

Answer detail: What will you do next?


The correct answer is b.
a. Explain him about the
This patient is asymptomatic and danger of driving associated
generally well. with his condition
In Australia, it is recommended to b. Tell him to surrender his
check fasting blood glucose every driving license now
3 years for non-diabetic c. Prescribe modafinil
individuals over the age of 40. So d. Inform local driving licensing
this patient would need fasting authority
blood sugar tested in 3 years, not e. Advice to lose weight as this
10 years, even though his blood will reduce the risk of
sugar is normal now. Option a is accidents
incorrect.
Answer detail:
All adult over the aged of 45 and The correct answer is d
over should have 5-yearly
estimation of serum cholesterol. This patient has developed clinical
This is correct for this person features of narcolepsy and
(option b). possible obstructive sleep apnea.
He is legally not allowed to drive
Checking vitamin D level and until further assessment by sleep
vitamin B12 and red cell folates is physician is completed and
not recommended as a part of compliance is recorded.
general health screen in y
Young people before the age of As this patient is declining to see
60. These should only be the sleep physician and is not
requested if patient is at high risk ready to stop driving, it is medical
for these deficiencies. practitioner’s moral obligation to
notify the local driving licensing
Blood pressure should be recorded authority.
every 1-2 years on all people 16
years and over. So option e is Narcolepsy is present in 0.05% of
incorrect. the population and usually starts
in the second or third decade of
9. A 40-year old male truck driver life 26 sufferers present with
comes to your office with his wife excessive sleepiness and can have
for assessment. His wife is worried periods of sleep with little or no
about him as he snores every night warning of sleep onset. Other
symptoms include cataplexy, sleep What will you do next for the
paralysis and vivid hypnagogic recipient?
hallucinations.
a. Give immunoglobulin
Diagnosis of narcolepsy is made b. Give hepatitis B vaccine and
on the combination of clinical immunoglobulin
features, and multiple sleep c. Counselling and follow up
latency test (MSLT) with d. Lamivudine
diagnostic sleep study on the prior e. Give hepatitis B vaccine
night to exclude other sleep Answer detail:
disorder and aid interpretation of The correct answer is c.
MSLT.
This health care worker had the
They should have a review at least needle stick injury and blood test
annually by their specialist. came back positive for hepatitis B
surface antigen.
Those with narcolepsy perform
worse on simulated driving tasks This health care worker should be
and are more likely to have counseled about the result and
accidents than control subjects. should have followed-up testing at
three months and six months. If
A conditional commercial license the blood test remain positive for
may be granted by the driving hepatitis surface antigen beyond
licensing authority, taking into six months, refer the patient to a
account the opinion of a specialist hepatologist.
in sleep disorder, and the nature
of the driving task and subject to Lamivudine is an antiviral drug
periodic (at least annual) review, that can be used for the treatment
after the following requirement of chronic active hepatitis
are met. however it is not used for the
treatment of acute hepatitis B.
Patient should be made aware of
the effect of their condition on As this recipient has already
driving and should be advised of developed hepatitis B infection
their legal obligation to notify the already from needle stick injury,
driver licensing authority where the role of vaccine and
driving is likely to be affected. The immunoglobulins is limited.
practitioner may themselves
advise the driving licensing 11.You are working as a resident in a
authority as the situation hospital. One of your colleagues is
requires. intoxicated with alcohol before
starting morning ward round.
10.A junior doctor in the hospital had
a needle stick injury while setting a. Notify medical board
up the line for intravenous fluids. b. Ask him not to drink alcohol
The patient status for hepatitis B is in future
unknown. The recipient has no c. Notify nurse unit manager
history of previous hepatitis B d. Notify chief executive officer
vaccination. of the hospital
The blood test of the recipient e. Take hand over from him
came back positive for HbsAg. and tell him to go home

Answer detail:
The correct answer is a. 4-the person follows medical
advice, including adherence to
Intoxication by alcohol or drugs medication.
while practicing or training in
profession constitutes misconduct. It is the responsibility of the
This essentially notifiable to the driving license holder to notify the
medical board. police about the conditions
applied due to medical illness.
12.A 39-year old lady presented with
a second episode of seizure within 13.A 22-year-old university student
one week. An EEG confirmed the presents for HIV checkup as she
diagnosis of epilepsy. She has been had unsafe sex with her new
started on carbamazepine. boyfriend not known to have HIV 2
days ago. You performed the test
She asked you when she can and report comes back negative.
drive? What will be the next step?

a. Within 12 months after a. Call the police to arrest the


review unknown person
b. Within 4 weeks b. Liase on with local council to
c. She cannot drive as long as trace the unknown person
she is on carbamazepine c. Perform HIV test in 12
d. She can drive as long as she weeks
is on carbamazepine d. Start her on ciprofloxacin for
e. Call the police and notify 14 days
them to cancel her license e. Reassure her that everything
is fine
Answer detail:
The correct answer is a.
Answer detail:
If a patient has been diagnosed The correct answer is c.
with epilepsy for the first time, a
conditional license may be It takes up to 12 weeks before
considered by the driver licensing HIV is detected in the blood. This
authority subject to at least is called the window period.
annual review in 12 months,
taking into account information If someone has unsafe sex, it is
provided by the treating doctor as required to wait for 12 weeks
to whether the following criteria before a test can reliably confirm
are met: or rule out HIV infection.

1-the person has been treated for During this time patient need to
at least 6 months. practice safe sex and avoid blood
donation.
2-there have been no seizures in
the preceding six months Contacting police or local council
is not required. Starting her on
3-if any seizures occurred after ciprofloxacin is not required.
the start of treatment, they
happened only in the first 6 Ceftriaxone and azithromycin may
months after starting treatment be needed if Chlamydia
and not in the last 6 months. trachomatus and Nesseria
gonorrhea infection is suspected.
Patient should also be tested for Answer detail:
hepatitis B and hepatitis C The correct answer is a.
infection.
In Australia, food allergy is the
If any of the test comes back commonest cause of allergic
positive, partner should be traced reactions in children is responsible
and tested for sexually for about 80 percent of
transmitted infections as well. anaphylactic reactions in which
the cause has been identified.
14. A 55-year old man presented for
evaluation of his oral cavity. He is It should be noted that drugs are
heavy smoker and heavy drinker. the most common cause of
He also chews tobacco and lives anaphylaxis overall including
alone. Previous advice to change children and adults.
life style has failed. Examination of
oral cavity is negative for any ulcer In adults, foods are implicated in
or lesions. only 20-30% of cases of allergy.

a. Refer him to dentist 16. A 60-year old male has been


b. Follow up in 12 months recently diagnosed with type II
c. Follow up in 24 months diabetes mellitus and his blood
d. Discharge from the clinic sugar came under control with
e. Inform him to see you if change in diet and exercise. He is
develops in problem in next a commercial driver and is keen to
3 months know if he can drive his vehicle on
highways.
Answer detail:
The correct answer is b. What is the most appropriate
advice?
A person who is elder than 50
years of age and his heavy smoker a. No restriction for driving
and heavy drinker is at risk for applies
developing cancer of the oral b. Conditional driving license
cavity. with annual review
c. Conditional driving license
Even though clinical examination with 6-monthly review
is normal, it is recommended to d. Conditional driving license
organize examination of the oral with 2-yearly review
cavity every 12 months. So option e. Unfit for driving until
b is correct. reviewed by endocrinologist

All other options are incorrect. Answer detail:


The correct answer is a.
15. Which of the following is the
ommonest cause of allergy in Patients diagnosed with type II
children in Australia? diabetes mellitus, controlled on
diet has no restriction on driving
a. Food allergy commercial as well as private
b. Seasonal allergy vehicles.
c. Pollen allergy
d. Insect venom Patients diagnosed with diabetes
e. Medication mellitus managed with insulin
may obtain a conditional driving
license subject to annual or 2- 18. A 14-year-old male attended your
yearly review clinic with his mother. The child
has history
Review by endocrinologist is not of allergy to yeast and asthma.
require for fitness to drive in Mother wanted her son be
diabetes. vaccinated for human papilloma
virus infections before becoming
17. A 44-year-old women returned sexually active.
from India 3 weeks ago. She has
been feeling nausea, vomiting, What will be the most appropriate
mild abdominal pain and diarrhea. advice?
Stool microscopy and culture
confirmed human roundworm a. HPV vaccine cannot be given
infestation. now due to history of
asthma
Which ONE of the following is first b. HPV vaccine cannot be given
line option for treatment? due to history of yeast
allergy
a. Pyrantel c. HPV vaccine can be given as
b. Mebendazole he is not sexually active yet
c. Albendazole d. Consider this issue on next
d. Ivermectin appointment
e. Metronidazole e. Refer this case to infectious
disease specialist
Answer detail:
The correct answer is a. Answer detail:
The correct answer is b.
This patient has clinical features
consistent with Ascaris A child should not be given HPV if
lumbricoids infestation. Patient he/she has history of yeast
usually reports the symptoms of allergy. Also HPV vaccine should
infestation and diagnosis is be avoided in pregnancy, in people
confirmed finding eggs in the with bleeding disorders and in
feces cases of anaphylaxis to the
vaccine or any of the vaccine
Patient may develop respiratory ingredients.
tract symptoms or gastrointestinal
symptoms and may notice 19. An 82-year-old woman visited you
roundworms when they emerge for the first time in your clinic after
from anus. the death of her husband. She
First line option is Pyrantel at a never saw a doctor in last 20
single dose or repeat it after 7 years. There is no documented
days in cases of heavy infestation. medical history. She is mildly
depressed however she is coping
Albendazole and Mebendazole are well otherwise/ physical
first line options to treat examination is normal.
hookworm (ankylostomiasis)
infestation. Ivermectin is used to What will you offer at your clinic on
treat human threadworm this visit?
(strongyloides) infestations.
Metronidazole is used for a. Pap smear
treatment of amebiasis. b. Mammography
c. Bone scan Debridement and wound wash is
d. Blood pressure required along with intravenous
measurement antibiotics as initial step to treat
e. Urine microscopy and this wound.
culture
How will you proceed regarding
Answer detail: consent?
The correct answer is d.
a. No consent required as
This patient is an elderly and has patient is already involuntary
been reluctant to visit a doctor for b. Take consent from the
20 years. The first step is build treating psychiatrist
rapport with the patient and doing c. take consent from the patient
a very thorough examination d. Contact the family of the
(especially checking blood patient for informed consent
pressure). This is key in building e. Apply to the mental health
therapeutic relationship with the tribunal for consent to debride the
patient. wound

Once a good rapport is build, Answer detail:


options of mammography and pap The correct answer is c.
smears should be discussed.
Recommended age for both Pap Patients who are subject to an
smear and mammography is up to involuntary treatment under
70 years however this patient may mental health act(Australia), may
choose to have these on later still have capacity to give consent
appointment. or decline consent for a specific
healthcare matter not related to
Also the risk of cardiovascular their mental illness (for example
disease increases with age and, the use of antibiotics for chest
risk of cervical and breast cancer infection, or surgical procedure on
does not increase after the age of the limb).
70 years, all screening test for
cervical and breast malignancy are An assessment of the patient’s
recommended up to the age of 70 capacity to make decisions about
year. All screening test for cervical health care for a specific matter is
and breast malignancy are undertaken in the usual way and
recommended up to the age of 70. documented appropriately.

Bone scan is not a screening test If patient lacks capacity to give


is not required u8ntil she is found consent for health care for a
to have fracture. Urine condition unrelated to mental
examination is not required as she illness, the consent of the
is asymptomatic. substitute decision maker is
required enabling them to be
20. A 25-year old male has been treated for that condition outside
admitted to mental health ward the mental health act.
under involuntary treatment order
for psychosis. On physical Involuntary treatment order only
examination, he is found to have applies to mental treatment.
5cmx5cm ulcer on the plantar
surface of the left foot. For any other medical condition,
consent should be sought from the
patient. Most mental health The Australia absolute risk
patients can give consent for any calculator combines several risk
medical treatment if they have factor to calculate a risk score
capacity. (expressed as a percentage),
which is a person’s chance of
21. A 40-year old woman comes to having a CVD event such as a
your office for advice. Her mother heart attack or stroke in the next
had MI a few years ago, and her five years.
father has a history of
hypercholesterolemia and is on 22. A 30-year old woman is concerned
statins. about breast cancer has her
Her blood pressure is mother was diagnosed with breast
150/90mmHg, fasting cholesterol cancer at the age of 48. Clinical
is seven mmol/l, HDL is 1.0mmol/l, examination of the breast is
and fasting blood glucose is 5.6. normal.
She is a non-smoker.
What is the most appropriate
What would you do next? advice for her?

a. Review in 3 months a. Mammography every year


b. Advice healthy diet from the age of 40
c. Regular exercise b. Mammography every 2 years
d. Calculate absolute from age of 50-69
cardiovascular risk factor for five c. Refer to surgeon
years d. Refer for genetic studies
e. Start with simvastatin and e. Reassure her that she is not
review in 6months at high risk of breast cancer

Answer detail: Answer detail:


The correct answer is d. The correct answer is a.

This patient’s clinical information If a woman has a first degree


should be used to calculate the relative less than 50 years of age
risk of cardiovascular disease in diagnosed with breast cancer, she
next five years. should have mammography every
Absolute cardiovascular disease year from the age of 40.
risk is the statistical probability of
a cardiovascular event occurring All other options are incorrect.
within a five-year period. It
reflects a person’s overall risk of 23. A 25-year-old male presented for
developing cardiovascular disease review in your office. His father
(CVD) replacing the traditional was diagnosed with colon cancer at
method that considers various risk the age of 35 and his uncle was
factors, such as high cholesterol diagnosed with colon cancer at the
or high blood pressure, in age of 50. His baseline
isolation. investigations including fasting
blood glucose, full blood count,
Clinical decisions based on renal function tests, liver function
absolute CVD risk improve health tests and iron studies are all
outcomes by identifying people normal.
most at risk and directing them to
the right treatment. What will you advise him?
a. Foecal occult blood test every but on different sides of the
6 months family.
b. Colonoscopy now
c. Colonoscopy every year Screening guidelines for this
d. Colonoscopy every 3 years category include:
e. No further action required
-FOBT is recommended at least
Answer detail: every two years for all people over
The correct answer is b. the age of 50. A full examination
of the large bowel, preferably by
This patient has one first-degree colonoscopy, is recommended in
relative with a history of colon those shown to have a positive
cancer at the age of 35 and FOBT. If eligible, participation in
another second-degree relative the national bowel cancer
has a history of colon cancer at screening program should be
the age of 50. His risk of recommended. For those not
developing bowel cancer is eligible for the national program,
moderately high and he should advice on access to FOBT is
have a colonoscopy 10 years available from The Cancer Council
before the onset of colorectal Helpline.
cancer in his father. So he should
have a colonoscopy now. -Consider sigmoidoscopy
(preferably flexible) every five
According to Cancer Council years from the age of 50.
Australia guidelines, Individuals
can be placed in one of three Category 2 — those at moderately
categories of relative risk and increased risk:
based on their family history.
This covers one to two percent of
Category 1 — those at or slightly the population.
above average risk:
People fit into this category if
This covers about 98 percent of there is:
the population. People fit into this
category if there is: -One first-degree relative with
colorectal cancer diagnosed
-No personal history of colorectal before the age of 55 (without
cancer, colorectal adenomas or potentially high-risk features as in
chronic inflammatory bowel category 3); or
disease and no confirmed close
family history of colorectal cancer; -Two first or one first and one
or second-degree relative/s on the
same side of the family with
-One first-degree (parent, sibling, colorectal cancer diagnosed at any
child) or second-degree (aunt, age (without potentially high-risk
uncle, niece, nephew, features as in category 3).
grandparent, grandchild) relative
with colorectal cancer diagnosed Screening guidelines:
at age 55 or older; or
-Offer colonoscopy every five
-Two relatives diagnosed with years starting at 50 or at an age
bowel cancer at age 55 or older 10 years younger than the age of
the first diagnosis of colorectal
cancer in the family, whichever [suspected familial adenomatous
comes first. polyposis (FAP)]; or

-Flexible sigmoidoscopy plus -Member of a family in which a


double-contrast barium enema or gene mutation that confers a high
CT colonography may be offered if risk of bowel cancer has been
colonoscopy is contraindicated for identified.
some reason.
Screening guidelines:
-Consider offering FOBT in the
intervening years. Patients should -Consider referral to a familial
be informed that a positive test cancer service for further risk
will require further investigation. assessment and possibly genetic
testing.
Category 3 — those at potentially
high risk: -Refer to a bowel cancer specialist
to plan appropriate surveillance
This covers less than one percent and management. This may
of the population. include:

People fit into this category if Familial Adenomatous Polyposis:


there are:
Flexible sigmoidoscopy yearly or
-Three or more first-degree second-yearly, starting from age
relatives or a combination of first 12-15 years until polyposis
and second-degree relatives on develops, then prophylactic
the same side of the family surgery. If family genetic testing
diagnosed with colorectal cancer, is inconclusive and no polyposis
or develops, sigmoidoscopy reduced
to every 3 years after the age of
-Two or more first or second- 35, then change to population
degree relatives on the same side screening if examinations normal
of the family diagnosed with to age 55. Prophylactic surgery,
colorectal cancer plus any of the e.g. restorative proctocolectomy,
following high-risk features: is appropriate for those with
proven FAR.
1- Multiple colorectal cancers in a
family member. HNPCC:

2- Colorectal cancer before the Colonoscopy every one to two


age of 50 years. years from age 25, or five years
earlier than the youngest
3- A hereditary non-polyposis diagnosis in the family (whichever
colorectal cancer (HNPCC} related comes first). FOBT may be offered
cancer (endometnai, ovarian, in alternate years or to subjects
stomach, small bowel, renal pelvis unwilling to accept colonoscopy.
or ureter, biliary tract, brain There are options for surveillance
cancer): or at other sites, usually starting
from age 25-35. Prophylactic
- At least one first-degree or surgery may be appropriate for
second-degree relative with a some.
large number of adenomas
throughout the large bowel
24. Which of the following would friend at a shelter
increase the risk of breast cancer accommodation. She is at high
in post-menopausal women? risk of being sexually assaulted.
As she lives independently, so
a. Hypertension there is no need to involve child
b. Hyperlipidemia protection services.
c. Obesity
d. Breastfeeding up to six However, she should be treated as
months a minor with the capacity to make
e. Physical activity for 30 decisions.
minutes daily
She should have a forensic
Answer detail: examination by an appropriately
The correct answer is c. qualified trained doctor. She
should be offered emergency
Obesity is a risk factor for the contraception and STI checks. She
development of breast cancer in should be given prophylaxis for
post-menopausal women. STIs as well.
Hyperlipidemia and Hypertension,
both are not related to the She will need to have follow-up at
increased likelihood of breast 2, 6, and 12 weeks following STI
cancer. checks.

Exclusive breastfeeding for up to 6 26. You are working in a surgical ward


months and active lifestyle where a nurse reports to have
decreases the risk of breast needle-stick injury while putting a
cancer development. new cannula. Patient is HBsAg
negative and is unlikely to be in
25. A 15-year-old girl comes to your window period as noted from
office with a history of unwanted history.
sexual intercourse last night. She The nurse is not sure of her
lives with her same age friend at a hepatitis B immunization status.
shelter accommodation. You have
offered her emergency What should be next step in this
contraception and explained her situation?
about STIs (sexually transmitted
infections) in details. a. No further follow up needed
b. Give hepatitis B
What will you do next? immunoglobulins and hepatitis B
vaccine
a. Referral to sexual assault c. Organize full course of
counsellor hepatitis immunization
b. Inform child protection d. Report the incidence to
services infectious disease specialist
c. Inform parents e. Do blood test to check HBsAg
d. Notify police status in three months
e. Reassure
Answer detail:
Answer detail: The correct answer is c.
The correct answer is a.
If the source is HBsAg-negative,
This patient is a 15-year-old girl and unlikely to be in the window
who is living with her same age period, no further follow-up
testing is required of source or This patient is worried about his
health care worker (health care family history of prostate cancer.
worker should be sure of her Also, his age falls him the
hepatitis immunization status). moderate risk category for
prostate cancer.
However this nurse should have
full HBV immunization as she is He should have the digital rectal
not sure of her immunization exam and then he need to have
status. It should be commenced prostate specific antigen level
as soon as possible, preferably checked. He should be explained
within 24 hours. about the outcome of elevated
PSA as well.
Hepatitis B immunoglobulins are
not indicated in this situation. If the prostate specific antigen is
checked after rectal examination
Reporting the incidence to team for prostate, there is a possibility
leader or supervisor is encouraged of acute elevation in PSA level. So
however involving infectious after the digital rectal exam, PSA
disease specialist is not required. should be checked after 2 weeks.

Follow HBsAg testing again within 28. A 49-year-old married male is


3 months is needed if source is brought to the emergency
hepatitis B positive. department with internal bleeding
from injuries sustained in a motor
27. A 52-year-old man, otherwise vehicle crash.
healthy, comes to your clinic
concerned about his family history The patient has been stabilized but
of prostate cancer. requires a blood transfusion to
replace lost blood and to prepare
His father was diagnosed with for surgery.
prostate cancer at the age of 75
and a friend was diagnosed with When approached to consent for
the condition recently. the transfusion, the patient
refuses, stating that it is forbidden
He is worried he might get prostate by his religion. The attending
cancer. physician calls for a psychiatric
consult. The psychiatry fellow on
On digital rectal examination, his call proceeds to the emergency
prostate was smooth, soft and no room.
other abnormality noted.
Which of the following is the most
What would you do next? appropriate first step in evaluating
this patient?
a. Prostate specific antigen level
after 2 weeks a. Assess the patient's decision-
b. Ultrasound making capacity to refuse the
c. Annual rectal exam transfusion
d. Reassurance b. Call the patient's wife to take
e. Prostate biopsy consent for transfusion
c. Consult guardianship court
Answer detail: for consent
The correct answer is a. d. Order the transfusion against
the patient's wishes
e. Talk to the patient about patient and doctor regarding
blood substitutes and ask him if he intervention.
would consider
accepting one 29. A young woman with family history
of Huntington disease is keen to
Answer detail: know the chances of inheritance of
The correct answer is a. the disease in her children. She
appears normal with no abnormal
According to Australian law any movements. What is the likelihood
person who is an adult and has of genetic transmission of
decision-making capacity, the Huntington's disease in her case?
doctor should respect his /her
beliefs and should not intervene to a. No chance of transmission
impose treatment. b. 25%
c. 50%
As the here patient is refusing a d. 75%
life-saving intervention, the first e. 100%
important thing is to assess his
decision-making capacity that Answer detail:
includes understanding, The correct answer is c.
appreciation and reasoning.
Huntington disease is caused by
Once the decision-making capacity expansion of the cytosine-
is evaluated, then we will discuss adenine-guanine (CAG)
other blood substitutes like trinucleotide repeats in the HTT
hemacil and other intravenous gene located on chromosome
fluids to save his life. 4p16.3 called HD gene. Expansion
of the CAG repeat number over
The doctor cannot transfuse successive generations causes an
against patient’s wishes as it is earlier and more severe
considered as an assault on their phenotype, termed anticipation.
physical, religious and emotional
feelings. Huntington's disease is inherited
However, this rule does not apply in an autosomal dominant fashion.
to children who need an urgent The probability of each offspring
transfusion, and their parents are inheriting an affected gene is
against transfusion even they 50%. Inheritance is independent
belong to Jehovah's witnesses. of gender, and the phenotype does
not skip generations. A child born
The wife cannot give consent here to a person who carries the
unless the patient is unconscious Huntington's disease gene has a
or lack the capacity to make a 50 per cent chance of inheriting
decision and she has power of the gene and developing the
attorney or advance directives in disease.
favor of transfusion.
If a child does not inherit the HD
Guardianship court is consulted gene, he or she will not develop
when the patient is unable to the disease and cannot pass it to
make his decision like being subsequent generations.A person
unconscious and he is holding no who inherits the HD gene, and
power of attorney or advance survives long enough, will sooner
directive, and there is a conflict or later develop the disease.
between family members of the
So in this situation, the risk of
transmission of Huntington's How will you proceed?
disease would be 50%. She is
young and without symptoms a. Refer to a gynaecologist
however she may develop b. Obtain a court order
symptoms latter in her life. So it c. Try to convince her that she
would be the duty of care to offer should not proceed with this
her genetic testing to confirm her d. You tell her that you cannot
inheritance of disease. help her
e. Mother's consent is valid as
30. A 13-year-old girl presented to she is her guardian
your clinic for health check-up. She
disclosed she had unwanted sex Answer detail:
last night. She does not live with The correct answer is b.
her parents and is sexually active.
What will you do next? In circumstances where a young
person with an intellectual
a. Report police disability, is incapable of giving
b. Report to the parents consent, a procedure such as,
c. Visit shelter sterilization, cannot be carried out
d. Report to child protection lawfully without the authority of
agency Family court or Guardianship
e. Review in 2 days board in Australia.

Answer detail: Parents cannot consent for this


The correct answer is d. special medical procedure. So
obtain a court order before putting
Child sexual abuse occurs when a a referral to gynecologist.
child has been exposed or
subjected to sexual behaviors that 32. Which of the following 'cancer risk'
are exploitative and/or is not increased in obesity?
inappropriate to his/her age and
developmental level. a. Hepatocellular cancer
b. Breast cancer in post-
Examples include sexual menopausal women
penetration, inappropriate c. Colorectal cancer
touching, and exposure to sexual d. Esophageal cancer
acts or pornographic materials, or e. Renal cell cancer
using the internet for soliciting
children for sexual exploitation. Answer detail:
The correct answer is a.
In practice, all reports are made
to the Department for Child There is convincing evidence that
Protection's Mandatory Reporting overweight and obesity are risk
Service. They can be contacting factors for following cancers:
anytime of the day or week.
-Colorectal cancer
31. A mother requests you to organize -Renal cell carcinoma
hysterectomy for her daughter -Pancreas cancer
with developmental disability. -Cancer of oesophagus
Daughter is 18-year old and she -Endometrium and breast (in post-
cannot give the consent herself menopausal women)
due to intellectual disability. -Gallbladder cancer.
E-More than 15mm-highly
There is no evidence that obesity significant for normal people and
increase the risk of liver cancer. It shows active tuberculosis.
is worth noting that excess body
fat probably decreases the risk of The BCG vaccine should be given if
pre-menopausal breast cancer. the induration is less than 5mm.
Do not give BCG if induration is
33. A mother brings her 9-month old more than 5 mm. This child will
child, generally healthy, for BCG need a chest X-ray and further
vaccination. The family belongs to assessment of tuberculosis.
India and have been living is
Australia for 2 years. There is no 34. A 32-year-old woman comes to
history of active tuberculosis in the your clinic for advice regarding
family. You performed the screening. She has history of
tuberculin test which showed gestational diabetes during
induration of 10 mm in 72 hours. pregnancy 2 years ago. Her blood
sugar was normal 6 months ago.
What is the next best step? Her grandfather died of myocardial
infarction at the age of 80 years.
a. Give BCG vaccine now u Her sister has been diagnosed with
b. Repeat tuberculin test in diabetes mellitus at the age of 45.
three week time
c. Give BCG vaccine in a week What will you advise her?
time
d. Refer to Immunologist u a. Serum cholesterol every 5
e. Inform the mother that BCG years
should be avoided in this situation b. Fasting blood glucose every 3
years
Answer detail: c. Colonoscopy every year
The correct answer is e. d. Exercise tolerance test every
year
A tuberculin test should be e. FOBT every year
performed prior to BCG
vaccination in all individuals over Answer detail:
6 months of age. It takes about The correct answer is b.
48-72 hour to show the reaction.
It is not a good test to diagnose Fasting blood glucose every 3
TB. years, should be considered at any
age in following high risk groups.
If area of induration is:
A-Less than 5mm-no tuberculosis. 1- All people with a history of
previous cardiovascular event
B-5-10mm-typical of past BCG including acute myocardial
vaccination. infarction and stroke.
2- All the women with a history of
C-More than 5mm is significant if gestational diabetes mellitus.
patient is immunocompromised. 3- Women with polycystic ovary
syndrome.
D-More than 10mm-positive for 4- Those on antipsychotic drugs.
either positive or latent 5- Those with impaired glucose
tuberculosis. tolerance test.
This woman fall in this high risk
category and so does need fasting Even a doctor is not allowed to re-
blood glucose test once every 3 prescribe to himself an anti-
years to make early diagnosis of psychotic medication which might
diabetes mellitus. have been already prescribed by
psychiatrist even.
A patient without any family
history of colorectal cancer and 36. A 45-year-old woman is concerned
personal history of ulcerative about her risk of getting melanoma
colitis and colon cancer should as one of her cousin was recently
have FOBT at least every two diagnosed with melanoma. She has
years after the age of 50. fair complexion and has developed
multiple dysplastic nevi on arms
All adults aged 45 and over should and legs. She also had history of
have 5 yearly check of serum multiple episodes of sunburn in her
cholesterol for screening of childhood.
hypercholesterolemia.
Which ONE of the following would
35. You find one of your colleagues, make her the highest risk for
prescribing antipsychotics for himself. developing malignant melanoma?

What is the most appropriate a. Diagnosis of melanoma in


action that you will consider? cousin
b. Fair complexion
a. Advice your colleague against c. Multiple dysplastic nevus
self-prescribing d. Multiple episodes of sunburn
b. Report to the department in childhood
head e. All of the above
c. Report to the medical board
d. Do nothing as it is his Answer detail:
personal affair The correct answer is c.
e. Write an incident report
The presence of multiple
Answer detail: dysplastic nevi in this patient puts
The correct answer is c. her at high risk (6 times normal)
of developing malignant
Self-prescription of anti-psychotic melanoma in future.
medication by a doctor means
your colleague is either treating All other features (fair
psychosis or bipolar disorder by complexion, diagnosis of
himself. melanoma in second degree
relative, multiple episodes of sun
This may be assumed that he/she burn in childhood) increase the
might have impaired judgement risk of developing malignant
because of psychosis. melanoma between 2-5 times
normal.
So it is obligation to report the
matter to medical board of 37. Which of the following is not a
Australia for safety of both the symptom of nicotine dependence?
patients and the doctor.
a. Smoking immediately after
The board then will initiate an waking up
inquiry. b. Smoking when ill
c. Difficulty stopping smoking She was initially assessed in the
d. Finding the first cigarette of emergency department and was
the day the most difficult to give up found to be minimally
e. Smoking more in the responsive. However, after an
afternoon than in the morning hour or so she became
combative and was confused
Answer detail: and distressed. She was
The correct answer is e. sedated.
At about midnight, she was
Characteristics of smokers with moved to a respiratory ward.
nicotine dependence include: You are the attending doctor
1. Smoking soon after waking up. and considering her irreversible
2. Smoking when ill. condition, you plan to sign her
3. Difficulty stopping smoking. acute resuscitation plan for NFR
4. Finding the first cigarette of the (not for resuscitation).
day the most difficult to give up. This patient has no advance
5. Smoking more in the morning health directive signed. Her only
than in the afternoon. next of kin available in the
So e is correct response here. hospital is her daughter to
38. Which one of the following helps in communicate.
prevention of sleeping sickness? Which one of the following is
a. Avoid areas with tsetse files correct regarding this situation?
b. Vaccine a. It is purely a clinical
c. Careful with drinking water decision to decide about not for
d. Antibiotics resuscitation considering
e. Pentamidine her terminal illness

Answer detail: b. Consent is not needed to
The correct answer is a. with-hold or withdraw a life-
sustaining measure, when
Human African trypanosomiasis providing that treatment would
also known as sleeping sickness, be inconsistent with good
is caused by protozoan parasites medical practice
transmitted via the bite of a tsetse
fly. c. Consent to health care
There are 2 major methods to cannot be obtained from a substitute
prevent transmission of decision maker on behalf
trypanosomiasis: of the patient like her daughter
-Vector control-avoiding areas
with tsetse flies. d. Consider to make a
-Surveillance and early treatment decision by involving one of the
of identified cases. relatives who could
Pentamidine is used in the consent or object to" as further
treatment of trypanosomiasis intervention was
(sleeping sickness).There is no medically futile”
vaccine available for human
African trypanosomiasis e. A family member
experiencing anticipatory grief is
39. An 85-year-old female is unlikely to be a good
admitted to hospital with a history decision maker about a dying patient
of pulmonary fibrosis and chronic
respiratory failure on a Saturday Answer detail:
evening. The correct answer is d.
What will be your next step in
In this situation decision to management?
with-hold futile treatment should
be made after discussion with a. Aortic ultrasound
the daughter. b. CT anglogram
c. MRI scan
It is an offence under the d. No action needed until
Guardianship and Administration he is above 65
Act 2000 to carry out e. Refer to vascular
health care (including the surgeon
withholding of
health care) without obtaining Answer detail:
consent. The correct answer is a.

Consent is needed to withhold Physical examination is less


or withdraw a life-sustaining accurate and has low
measure, even if providing that sensitivity for the detection of
treatment would be abdominal aortic aneurysm
inconsistent with good (AAA).
medical practice.
It is recommended to perform
If the patient does not have a screening aortic ultrasound
an advance health directive, for detection of AAA in
consent to health care must asymptomatic patients with
be obtained from a substitute family history of AAA before
decision maker on behalf of age of 55 in men and 60 in
the patient (GAA, s. 66). women.

In this case, consent from Aortic ultrasound should be


patient's daughters is required used for screening of AAA
before not-for- after age of 65 in men and
resuscitation order could be over 70 in women without
made. family history of AAA.

It is possible that a family Ultrasound is very accurate in


member experiencing detecting an AAA. There is no
anticipatory grief is likely regular screening program for
not to be good decision maker AM in Australia yet.
about a dying
patient however it is legal 41. A 24-year-old male presented to
requirement to involve and your office for evaluation. He was
consent from family bitten by another person in a cafe
member before signing off any at night. The infectious status of
not-for- the victim is unknown. You have
resuscitation order. ordered blood tests for HIV, HCV
and HBV. However, the patient
40. A 50-year-old male presents declined to be tested for HCV and
with family history of abdominal HIV, however, is happy to be
aortic aneurysm. He is otherwise tested for HBV.
fit and health man with no other
symptoms. Cardiovascular You are contacted by the GP
examination is unremarkable. of the other person next
morning. The GP wanted to
know if you have requested However, testing for HIV and
the blood tests to check for HCV is not mandatory in
blood borne diseases. The Australia and this patient
victim is unwilling to give cannot be forced to undergo
consent for testing his blood tests.
hepatitis C and HIV.
In this situation, source
What will you do next? (victim) should be considered
unknown and biter should
a. Get the public health receive post-exposure
unit force victim to give her prophylaxis anti-HIV
consent medication by his GP along
b. Ask the GP to treat the with blood tests for HIV, HBV,
biter prophylacticaly HCV and regular follow up.
c. Order the laboratory to
perform the test without Ideally post-exposure
patient’s knowledge. prophylaxis for HIV in high-
d. Do nothing assuming risk patients (where the
victim is not carrying hepatitis source is unknown) without
C and HIV virus waiting for the blood test
e. Ask the biter to contact result as soon as possible
the victim and request and (not later than 72 hours).
convince him to give
consent for testing. 42.A 56-year-old man presents
to your clinic for advice about
Answer detail: Quitting smoking. He has a
The correct answer is b. history of depression.

This patient was bitten on the Which of the following is the


arm by an individual and the best option you can offer him
biter was exposed to the to help quitting
victim's blood.
a. Varenicline
The GP of the biter contacted b. Bupropion
you to add on hepatitis C and c. Nicotine replacement
HIV test as well to know the therapy
status of the source (victim). d. Nortriptyline
e. No medication needed
Ideally, the victim should be
consoled about the possibility Answer detail:
of transmission of blood The correct answer is b.
borne viral infections (HBV,
HCV, HIV) from the victim to Varenicline is a nicotine
the biter. partial agonist.lt can cause
neuropsychiatric adverse
Extensive counselling should effects and people taking
be done and information varenicline should be
should be provided in detail monitored for unusual mood
about the possibility of changes, depression,
positive HIV test and post- behaviour disturbance and
exposure prophylaxis for the suicidal thoughts.
biter.
Bupropion is a non-nicotine coronary heart disease,
oral therapy with similar smoking cessation is
efficacy to nicotine associated with a 36 percent
replacement therapy in aiding reduction in the risk of all-
smoking cessation. It is cause mortality. Wearing a
effective for smokers with properly adjusted seat belt
depression, cardiac or reduces the risk of mortality
respiratory diseases, and also by 50 percent. However this
to improve short-term mortality is related to trauma
abstinence rates for people and severe injuries and is not
with schizophrenia. due to cardiovascular disease.
Also advise the patient to
Nicotine replacement therapy reduce alcohol intake to a
and nortriptyline are second maximum of two standard
line options for smoking drinks per day and have at
cessation. least two alcohol-free days
per week. If patient is female,
43. A 60 year-old male visited she should take up to one
your clinic for routine medical standard drink per day with
checkup. He smokes 45 two alcohol-free days per
cigarettes per day, drinks week. Blood pressure and
alcohol 10 standard drinks body weight have direct
per day and his body mass association with each other.
index is 32.He wears seat- One percent reduction in
belt 60 percent of the times weight lowers systolic blood
when he drives. He is pressure by 1mmHg.The
motivated to follow life style benefits of weight reduction’s
modification advice. Which of include decrease in insulin
the following is the most resistance and improvement
significant advice for in hyperlipidaemia. Regular
prevention of cardiovascular exercise increases high-
disease and premature death density lipoprotein
in this patient? cholesterol and reduces body
weight. This in turn reduce
a. Quit smoking insulin resistance and
b. Reduce alcohol intake systemic vascular resistance.
c. Wear seat-belt every Regular exercise has an
time independent cardio-protective
d. Reduce weight activity.
e. Regular exercise
44. A young female intern
Answer detail: accidentally pricked herself
The correct answer is a. with the needle while taking
blood sample from an HIV
Smoking is strongly positive patient.
associated with high risk of
cardiovascular disease. What would be the most
Quitting smoking is appropriate next step?
considered the most effective
lifestyle interventions for a. Give antiretroviral drugs
preventing cardiovascular for 4 weeks
disease and premature b. Test both patient and
deaths. ln a patient with doctor for HIV viral load now
c. Do not allow doctor to (lamivudine+zidovudine), or
draw blood for 3 months (emtricitabine+tenofovir).
d. Give her intravenous
immunoglobulins 45. A 48-year-old woman comes
e. Educate her that risk of to your office to discuss
disease transmission is very about breast cancer. Her
low friend has been recently
diagnosed with breast cancer.
Answer detail: She has no family history of
The correct answer is a. breast cancer and she has no
personal risk factors for
In the situation of needle breast cancer. She has no
stick injury and exposure to lump found in breasts on
an HIV positive person, post- clinical examination.
exposure HIV prophylaxis is
recommended. What is the most appropriate
management?
Provision of post exposure
prophylaxis should not be a. Refer her to breast
delayed while establishing screening now
the source status. Early b. Consider a 6 monthly
initiation of post-exposure clinical review
prophylaxis (PEP), as soon as c. Do yearly ultrasound
possible after exposure, is d. Teach how to do self-
strongly recommended. PEP breast examination
should not be offered more e. Commence 2-yearly
than 72 hours after exposure. mammogram from her age of
50
A 28-day course of PEP is
recommended. This health Answer detail:
care worker should have The correct answer is e.
follow-up HIV antibody
testing at baseline, 6 weeks This woman is not at high risk
and 3 months, and up to 6 for developing breast cancer
months, along with tests for and her examination is
other blood-borne viruses as unremarkable. She should
above. have mammography, every 2
years from age 50 to 69
Recommendations for years.
occupational post-exposure
prophylaxis include a basic All other options are
regimen of two incorrect.
nucleoside/nucleotide reverse
transcriptase inhibitors for 46. A 7-year-old female child is
most HIV exposures, and an seen in your office for
expanded regimen with the unusual behavior. She
addition of a third drug when displayed sexual behavior
the exposure poses an with neighbor kids. This child
increased risk for is living with her foster
transmission. mother as her own mother
has been divorced.
Examples of antiretroviral
drug combinations include What will you do next?
The correct answer is b.
a. Notify child protection
services This medical student had
b. Get psychiatric needle stick injury and needs
evaluation post-exposure prophylaxis
c. Send a blood test for according to guidelines. The
HIV exposed person is not immune
d. Chase collateral to hepatitis B.
information about the family
structure So he should be given a single
e. Notify hospital dose of hepatitis B
administration immunoglobulin and three
doses of hepatitis B vaccine
Answer detail: over six months.
The correct answer is a.
Also, the status of the source
If a child is displaying sexual and exposed person should be
behavior inappropriate for followed up by requesting full
her/his age, there is high hepatitis B serology.
suspicion of child sexual
abuse. So immediately notify However, if the exposed
the child protection services person has previously been
and they will follow the case. vaccinated for hepatitis B, and
had a blood test documenting
All other options are incorrect a response to the vaccine,
as next step however would then no treatment is
be required after notification necessary.
to the child protection
services. All other options are incorrect.

47. A medical student sustained 48. A 47-year-old woman


needle stick injury while arranges a lengthy
getting intravenous access of consultation with her GP. She
a patient. The student is not complains of marital discord
immunized for the hepatitis with her husband due to her
B. The hepatitis B status of low sex drive. She has a lot
the patient is unknown. of things to discuss with the
GP. During the consultation,
What is the most appropriate the GP uses the sentences
management? like, "I'm listening', 'I
understand what you are
a. Immunoglobulins saying' or 'Go on' at
b. Immunoglobulins and appropriate times.
hepatitis B vaccine
c. Hepatitis B vaccine Which of the following
d. Request serology and interviewing techniques, the
wait for the result before GP is using?
commencing the vaccine
e. Reassure and consulting a. Confrontation
regarding safe occupational b. Open-ended Question
practice c. Closed-ended Question
d. Facilitation
Answer detail: e. Summarising
a. Tell her husband
Answer detail: b. Tell her husband GP
The correct answer is d. c. Do nothing
d. Notify the department of
Facilitation encourages health
communication by using e. Tell her husband once
manner, gesture or words that she leaves
do not specify the kind of
information that is being Answer detail:
sought. It suggests that the The correct answer is d.
doctor is interested and
encourages the patient to Chlamydia is a notifiable
continue. Silence and disease and it must be
facilitation go hand in hand. reported to the department of
health within 5 days of
Option a) is incorrect as the diagnosis.
technique of confrontation is
used whereby the interviewer It is legal requirement to
describes to the patient protect the public from harm.
something striking about his
or her verbal or non-verbal The department of health
behavior when the patient is team will devise a method to
not speaking freely or clearly contact possible contacts.
Examples are: You look sad',
You seem frightened' etc. Reassure patient that
confidentiality will be
Option b) is incorrect. maintained by using the name
Option c) is incorrect. code during contact tracing.

Option e) is incorrect. 50. A 35-year-old woman


Summarizing what the patient presented to your office for
has said can keep the patient breast cancer screening as
on track and help you to check one of her maternal aunts
the accuracy of the was diagnosed with breast
information by providing the and ovarian cancer at the age
patient with the opportunity of 40.
to revise any
misunderstandings, e.g., 'If What will you advise her?
I've understood you correctly
you have told me. a. 2 yearly mammography
b. 2 yearly ultrasound
49. A young female presented c. Genetic risk screening
with dysuria, vaginal d. BRCA gene screening
discharge and post-coital e. 6 monthly self-breast
bleeding. She was diagnosed examinations
with chlamydia. She was
happy to get her own Answer detail:
treatment however she The correct answer is d.
refused to tell her husband.
This patient is at high risk for
What will you do in this both breast and ovarian
situation? cancer due to positive family
history (second degree
relative) of breast and ovarian a. HPV vaccine not
cancer. indicated for males
b. Give him vaccine 2
Two genes, BRCA1 and BRCA2, years after he has been
are known to dramatically sexually active c. Give
increase a woman’s risk of him HPV vaccine now
developing breast cancer and d. Male of 12-13 year of
ovarian cancer in her lifetime. age is only eligible for HPV
vaccine
Only 5-10% of those breast e. Advise condoms as that
cancers are caused by an is the only way of preventing
inherited genetic mutation. HPV infection
The same gene can put women
at a higher risk of ovarian Answer detail:
cancer as well. The correct answer is c.

Genetic counseling for BRCA1 Human papilloma causes


and BRCA2 mutations can tell cervical cancer in women and
a woman whether she is at is transmitted through sexual
increased risk for these activity. A vaccine called
Cancers. Gardasil has been developed
that protects against the two
BRCA gene screening is high-risk HPV types (types 16
indicated in following and 18), which cause 70% of
situations: cervical cancers in women and
90% of all HPV-related
1- High numbers of family cancers in men.
members with cancer
diagnoses (especially breast It also protects against two
and ovarian) throughout low-risk HPV types (types 6
several generations, either and 11), which cause 90% of
maternal or paternal. genital warts.
2- Family member diagnosed
with cancer before age 50. All boys and girls aged 12-13
3- Family members who have years should have the HPV
been diagnosed with multiple vaccine. The vaccine is most
cancers (for example, breast effective at this stage, before
and ovarian). sexual activity has
4- Male breast cancers, or commenced and when the
clustering of other cancers body produces more
such as colon, prostate, antibodies.
stomach or pancreatic.
However if a male presents to
51. A 17-year-old male attended GP for HPV vaccination before
your clinic with his girlfriend. starting sexual activity, a
His girlfriend wanted him general practitioner can
vaccinated against human prescribe this vaccine in girls
papilloma virus before before age of 26 and in boys
becoming sexually active. before age of 21.

What would be the most All other options are not


appropriate advice? correct
52. You are a GP in a rural area, one of the most efficient ways
and you are actively involved to reduce the risk of dying
in bowel cancer screening of from bowel cancer.
your patients.
What of the following The FOBT can detect blood
statement is false about from pre-cancerous polyps or
bowel cancer screening? early stage bowel cancer.
When identified early, 90% of
a. An asymptomatic 48- bowel cancer cases can be
year old man with no family successfully treated.
history of colorectal
cancer does not need a Bowel cancer screening
screening for colorectal (FOBT) is recommended every
cancer two years from age 50-75
b. A 55-year old man with years and could be continued
no family should have a beyond 75 years depending on
faecal occult blood test individual circumstance.
(FOTB) every two years
If there is no personal history
c. A 55-year old man with of bowel cancer, colorectal
one first-degree relative with adenomas, inflammatory
colorectal cancer diagnosed bowel disease or family
at 60 years should have a history of bowel cancer or one
faecal occult blood test first-degree or second-degree
(FOTB) now and every two relative with CRC diagnosed
years aged above 55 years, the
patient would be at low
d. A 50-year old man with average risk of developing
one first-degree relative with bowel cancer and two yearly
colorectal cancer diagnosed FOBT should be continued.
at 52 years should have a
colonoscopy now and every If an asymptomatic patient
five years with has one first-degree
relative with bowel cancer
e. A 35-year old man with diagnosed aged below 55
one first-degree relative with years or two first-degree or
colorectal cancer diagnosed one first-degree and one
at 52 years should have a second-degree relative(s) on
colonoscopy now and every the same side of the family
five years with bowel cancer diagnosed
at any age, then the patient
Answer detail: would need colonoscopy from
The correct answer is e. the age of 50 or at an age 10
years younger than the age of
He should have a colonoscopy first diagnosis of CRC in the
every five years from 42 years family, whichever comes first.
of age (10 years younger than
the age of the first diagnosis 53. A 33-year-old Australian
of colorectal cancer in the female presented with her six
family. month old unvaccinated child.
She has read in the
Screening using a faecal newspaper that vaccines are
occult blood test (FORT) is not safe and she planned to
discuss the matter with you. increase in life expectancy
After discussion and of up to 10 years, if it occurs
understanding the early enough
significance of vaccines, she e. Smoking soon after
decided not to get her child waking is an indicator of
vaccinated. What is the most nicotine dependence
appropriate step you would
consider in this situation? Answer detail:
The correct answer is b.
a. Call the child protection
services Smoking is responsible for
b. Notify the police to about 20% of all cancer
arrest her and present before deaths in Australia. Macular
the court c. Inform the degeneration has a strong
local community council association with smoking.
d. Respect her decision
e. Apply for guardianship Indigenous Australians are 2.2
for the child times more likely to smoke
tobacco as compared to non-
Answer detail: indigenous Australians.
The correct answer is d.
There is a strong evidence
Vaccination is not compulsory that quitting smoking in early
in Australia a parent decides life increases life expectancy
not to get a child vaccinated, of up to 10 years. Nicotine
it should be respected.in this dependence is present when a
situation doctor has informed smoker smokes a cigarette
her about benefits and risks immediately on waking up as
associated with vaccines and first thing.
she understands everything.
So final decision should be 55. A woman had recently
made by the parents or single undergone a pap smear.
parent to get their childi- Which of the following types
3cciriated or not. Involvement of preventions she is involved
of child protection services or with?
police is not required.
a. Incidental prevention
54. Which of the following b. Secondary prevention
statement regarding smoking c. Primary prevention
in Australia is not true? d. Tertiary prevention
e. None of the above
a. Smoking is responsible
for 20 percent all cancer Answer detail:
deaths The correct answer is b.
b. Macular degeneration
has no association with Secondary prevention includes
smoking actions taken to stop or delay
c. Indigenous Australians the progression of the
were 2.2 times as likely as disease.
non- Indigenous
Australians to smoke tobacco The term is usually applied to
d. Successfully quitting measures for the detection of
smoking can result in an disease at its earliest stage,
i.e. in the presymptomatic There are some barriers in the
phase, so that treatment can communications such as long
be started before irreversible length of waiting time, sense
pathology is present. of urgency in the doctor, a
poorly lit room, a long desk in
The early recognition of between the physician and the
hypertension through routine patient, a poor record system.
testing (screening) of patients
allows treatment during the 57. In Australia, shared care
presymptomatic phase of the model of care is used for care
illness process and management of pregnant
women. Antenatal care is a
Examples include a pap smear, major component of care in
mammography and endoscopy General Practice.
for polyps of the large bowel.
Which of the following is true
Primary prevention includes regarding antenatal
action taken to avert the investigations in pregnancy?
occurrence of disease.
a. Syphilis serology should
Tertiary prevention includes be performed only on those
the management of at risk b. Hepatitis E
established disease so as to antibody level should be
minimize disability. carried out on all women
c. A glucose tolerance test
56. In a regular General should be conducted on all
Practitioner consultation in pregnant women in early
Australia, which of the pregnancy
following is an example of a d. Screening for risk of
communication barrier with Down syndrome should be
the patient? offered to all pregnant
a. Short length of waiting women
time e. Performing a Pap test in
b. Good record system pregnancy is contraindicated
c. An adequately lit room
d. A receptive demeanor of Answer detail:
the doctor The correct answer is a.
e. A Long desk in between
the physician and the patient Syphilis serology is performed
on all pregnant women.
Answer detail:
The correct answer is e. Option b is wrong as Hepatitis
E screening is not routinely
Effective communication done.
depends on four interrelated
factors concerning the Option c is wrong as glucose
message—the doctor (the tolerance test is offered
sender), the patient (the between 24-28 weeks of
recipient), the message itself pregnancy.
and the environment in which
the message is sent. Option e is incorrect as Pap
smear can be done in
pregnancy up to 28 weeks of
gestation. However it is Among caregivers, significant
usually recommended to have risk factors for elder abuse
pap smear after delivery if it are:
is not urgent.
- Inability to cope with stress
58. Elder Abuse is common in (lack of resilience).
Australia. Which of the - Depression, which is
following is likely to be the common among caregivers.
most common cause of elder - Lack of support from other
abuse? potential caregivers.
- The caregiver's perception
a. Weakening of the that taking care of the elder is
decision-making capacity of burdensome and without
the elderly person psychological reward.
b. Financial dependence - Substance abuse.
c. Caregiver degree of
stress 59. The free National Human
d. Medical condition in the Papillomavirus (HPV)
elderly patient Vaccination was introduced in
e. Social isolation 2007 in Australia.

Answer detail: What of the following


The correct answer is c. statement about HPV
Vaccination in Australia is
Although, all of the above are false?
potential risk factors for elder
abuse but among these, the a. All eligible girls should
most common cause of elder be encouraged to HPV
abuse is the degree of the Vaccination in the first
stress of the career. year of high school
b. All eligible boys should
The stress of elder care can be encouraged to HPV
lead to mental and physical Vaccination in the first
health problems that make year of high school
caregivers burned out, c. An HPV vaccinated
impatient, and more woman does not need
susceptible to neglecting or cervical cytology screening
lashing out at the elders in d. HIV is not a
their care. contraindication for HPV
Vaccination
Even caregivers in e. HPV types 16 and 18 are
institutional settings can associated with cervical
experience stress at levels cancer
that lead to elder abuse.
Nursing home staff may be Answer detail:
prone to elder abuse if they The correct answer is c.
lack training, have too many
responsibilities, are unsuited Participation in the Human
to caregiving, or work under Papilloma Virus (HPV)
poor conditions. Vaccination Program is
encouraged in all eligible boys
and girls in the first year of
high school.
Answer detail:
The HPV vaccine is provided The correct answer is e.
free in schools to all males
and females aged 12-13 years As per the guidelines, abused
under the National HPV women are usually
Vaccination Program. constrained from leaving
home by some factors. This
Current cervical cytology include:
screening recommendations
remain unchanged and should - Fear of reprisals- many
be followed regardless of the women are subjected to
HPV vaccination status. threats of injury and violence
to themselves or their children
The presence of if they leave. Approximately
immunosuppression, either 40-45% of women killed by
medically or in patients with their spouse are separated or
HIV infection, is not a in the process of separating.
contraindication for Gardasil®
or Cervarix®. However, the - Social isolation- several
immune response may be social factors contribute to
smaller in the why women feel they cannot
immunocompromised patient leave; having dependent
than in immunocompetent children, being deliberately
patients. isolated from friends and
family by the perpetrator, and
HPV types 16 and 18 are shame relating to injuries.
associated with cervical
cancer. - Financial dependence women
do not have equivalent
60. Domestic violence has shown earning capacity to men. To
an increasing trend in leave their partner condemns
Australia. Nearly, 17% of many women, and their
women aged 18 years and children, to a substantial
over had experienced decline in their standard of
violence by a partner. living.

Why may an abused woman - Emotional dependence and


remain in an abusive fear - many abused women are
relationship? committed to their
relationship, love their partner
a. She is financially and are hoping for a change in
dependent on her partner the relationship. Some abused
b. She loves her partner women are fearful that their
and fears what will happen to partner will not cope with a
the partner if she leaves separation and the partner
c. Her cultural or religious may be threatening to suicide
beliefs prohibit her from if she leaves.
leaving
d. Because of fear of - Poor self-esteem- after years
reprisals or counter charges of physical violence and verbal
from their partner abuse, many victims lose their
e. All of the above self-confidence and doubt
their ability to cope on their a. 2-yearly colonoscopy
own. from now
b. Mammography every
61. In Australia, which of the year
following behaviors does not c. Send her for genetic
constitute domestic/family studies
violence? d. Check blood sugar now
e. Request fasting lipids
a. Slapping and hitting
b. Sexual intercourse Answer detail:
against the will amongst The correct answer is a.
married couple c.
constant arguing without an Currently in Australia, it is
imbalance of power in the recommended to have 2-
relationship yearly colonoscopy for people
d. Stalking from age of 25-30 years if
e. Withholding money there is family history of
finances colorectal carcinoma:

Answer detail: -Three or more first or second


The correct answer is c. degree relatives with
colorectal carcinoma at any
Constant arguing without an age.
imbalance of power in the -Two or more first or second
relationship is not considered degree relatives diagnosed
a domestic violence in family with colorectal carcinoma
relationships. before the age of 50 years.
-A family member where
The forms of domestic genetic studies identify a high
violence include all of the risk.
other given options:
So this patient qualifies for 2-
-Sexual intercourse against yearly colonoscopy as her two
the will amongst the married family members have
couple. colorectal carcinoma before
-Slapping and hitting. the age of 50. So option a is
-Stalking. correct.
-Withholding money/finances.
All other options are incorrect.
62. A 30-year-old woman comes
to your clinic for advice 63. A 55-year-old female smoker
regarding screening. She is was prescribed nicotine patch
asymptomatic. Her mother by one of your colleagues on
and elder sister both were her weeks ago at your GP
diagnosed with colorectal clinic. The patient finds it
cancer at the age of 48. Her hard to quit smoking despite
father died at the age of 60 using nicotine patch. She
due to myocardial infarction. feels depressed as suicidal
Her grandfather was thoughts present. She wants
diagnosed with prostate your advice in this regard.
cancer at the age of 70. What What is the most appropriate
will you recommend her? advice?
a. Continue to use nicotine What would you do next?
patch only for few more
weeks a. Treat her, trace all her
b. Start her on bupropion contacts and treat them
and cease nicotine patch b. Advise to avoid sex with
c. Start her on bupropion too many people
while she uses nicotine patch c. Use condoms for 7 days
d. Cease nicotine patch and continue sexual activity
and ask her to come back in d. Trace only last contact
two weeks e. Refer her to and treat
psychiatrist e. Giver her antibiotics
last appointment 2 well
however no Answer detail:
The correct answer is a.
Answer detail:
The correct answer is c Contact tracing is an
important part of management
A study published in of Chlamydia. Patient name
Australian Prescriber showed and all identifying information
that bupropion is an option to related to patient are kept
treat depression associated confidential and are not
with quitting smoking. shared to any of his/her
contact.
The study revealed that
bupropion and nicotine when Patient is encouraged to give
combined are more effective details of all sexual contacts
in quitting smoking than in last 6 months (maximum
either of the medication alone. duration for Chlamydia tracing
in Australia) so that they can
While patient is smoking, be informed via phone or
bupropion can be started and letter to visit the sexual
nicotine patch is continued. health clinic as soon as
Cessation of smoking can be possible.
started in second week of
bupropion. About 68% of male partners
of infected women are found
If until seventh week, no to be infected with Chlamydia
cessation of smoking infection on contact tracing.
achieved, bupropion is less
likely to help then. All the sexual partners
exposed by oral, vaginal or
Frequent blood pressure anal sex without condoms are
monitoring is necessary with at high risk of getting
bupropion therapy. Chlamydia infection. They
should be treated regardless
Referral to psychiatrist is not of symptoms and test results.
required at this stage.
A patient who has been
64. A 21-year-old female with diagnosed with Chlamydia
multiple sexual partners for should not have sex with
last 6 months was recently anyone even using a condom,
diagnosed with Chlamydia. until 7 days after treatment is
completed.
retarded daughter at 8 weeks
Chlamydia is a notifiable of pregnancy. The mother
disease. Doctors and wants an abortion for her
laboratory staff are legally daughter as she is unable to
required to inform local health look after herself. The patient
authority in the state. does not want an abortion.

Treatment of Chlamydia is a How will you proceed?


single dose of 1-gram
Azithromycin or doxycyclin for a. Perform medical
7 days. Patient should not abortion as it is close to 9
have sex until at least a week weeks otherwise she will
has passed after the require surgical abortion
treatment. b. Do not perform abortion
as patient did not consent
65. A 30-year-old woman came and discharge home
for assessment of her c. Refer the matter to the
abnormal cervical pathology. family court
She had low grade squamous d. Get consent from the
intraepithelial lesion on Pap local council
smear 12 months ago and her e. Discuss the matter with
current Pap smear report also the father of the girl in a
shows possible low grade week
squamous intraepithelial
lesion. Answer detail:
The correct answer is c.
What will you do next?
This patient does not have
a. Cease further annual capacity to decide due to
screening mental disability.
b. Refer for colposcopy Her mother also does not have
c. Repeat Pap smear in 6 capacity to decide and consent
months for her daughter in this
d. Continue 2 yearly Pap particular situation as the
smears procedure is invasive and
e. Repeat Pap smear in 3 involves risks.
months
The matter should be referred
Answer detail: to either Supreme Court or
The correct answer is b. Family Court.

If a woman continues to have The Supreme Court can


2 consecutive pap smears exercise its role as the
showing low grade squamous supreme parent of children.
intraepithelial lesion (possible The Family Court of Australia
or definitive) at 12 months has a similar authority.
intervals, referral for
colposcopy is recommended. An application to the court
should be considered in
All other options are incorrect. situations, and with
procedures, that are so
66. A 52-year old mother brings serious that neither young
her 18-year old mentally person, parent nor can
guardian give valid consent. c. Fish oil
This includes situations d. Simvastatin
where: e. Ezetimibe

1. The procedure is very high Answer detail:


risk (for example, separating The correct answer is b.
conjoined twins).
Normal value of fasting serum
2. There may be life changing triglyceride is less than
effects (for example, 1.5mmo1/1.
sterilisation of mentally
disabled young persons, Elevated fasting triglyceride
abortions, removal of life level greater than 1.9 mmo1/1
support, the removal of increases the risk of coronary
organs for transplants, gender heart disease and stroke by
re-assignment and bone more than 30%.
marrow harvest). So this must be treated by life
style modification and fish oil.
3. There is a strong objection
from a dissenting parent. First-line treatment for
hypertriglyceridemia includes
4. A child with capacity to a diet rich in mono- and
make decisions is refusing polyunsaturated fat (option b)
healthcare and there is and low glycaemic index
significant risk of harm in carbohydrate food.
them doing so. Best modality to reduce
weight is caloric restriction
5. The procedure involves and exercise.
invasive, irreversible or major
surgery (excluding lifesaving The next step is consideration
emergency surgery). of marine omega-3 fatty acids
(fish oil) and fibrates.
If there is any doubt in
relation to consent to provide Simvastatin and ezetimibe are
healthcare to a child or young used to lower LDL cholesterol.
person, it is strongly
recommended that legal 68. A-54-year old male with
advice is obtained. history of COPD and
gallstones is undergoing
67. A 43-year-old lady comes to cholecystectomy. He still
your clinic for follow up of her smokes and manages without
blood tests you conducted on oxygen at home. What will be
last appointment. All of the your advice?
reports are normal except a. Abstain from smoking
fasting triglycerides level for 6 weeks before surgery
which is 2.0 mmo1/1. b. Abstain from smoking
for one week before surgery
Which of the following is next c. Discontinue smoking
step in her management? after surgery
d. Smoking has no effect
a. High glycemic index on the surgery
carbohydrate food e. None of the above
b. Poly-unsaturated fats
Answer detail: C-have you ever felt you
The correct answer is a. should cut down on your
drinking?
COPD is an independent risk A-have people annoyed you by
factor for major criticizing your drinking?
cardiopulmonary G-have you ever felt bad or
complications in major Guilty about your drinking?
surgical procedure. E- Have you ever had a drink
first thing in the morning to
Before surgery, patients with steady your nerves or to get
unstable COPD should receive rid of a hangover (Eye
oral steroids, which do not opener)?
compromise wound healing.
So only option d is from the
Patients with history of COPD CAGE Questionnaire and is the
are recommended to avoid correct response.
smoking for 6 weeks prior to
the surgery. 70. A 27-year-old male known to
be heroin-addict comes to
69. A 35-year-old male presented your clinic for a prescription
to your clinic. He has of diazepam as he is planning
background history of to kick of his heroin
depression and has been addiction. He is
taking alcohol regularly. He asymptomatic at the
wakes in early morning and movement.
feels nothing is interesting in
this world. What would be your course of
action?
What would you ask him
about alcohol intake? a. Prescribe him a dose of
diazepam for two weeks
a. Do you drink only in b. Tell him that diazepam
party is not used for this purpose
b. Do you drink on c. Prescribe him diazepam
everyday dinner time and he should collect his
c. Do you drink before tablet on daily basis
sleep from a pharmacy
d. Do you drink after wake d. Give him naloxone
up in morning e. Refer him to a drug
e. Do you invite your rehabilitation center
friends before having a drink
Answer detail:
Answer detail: The correct answer is e.
The correct answer is d.
This patient is motivated to
Any person with history of kick off his heroin addiction,
alcohol drinking should be so this opportunity should be
assessed for alcohol related used to refer him to drug
problems with CAGE rehabilitation center.
questionnaire:
In Australia, most
detoxification provided to
heroin-dependent persons is
provided in rehabilitation which reduces heroin
centers. Detoxification is often consumption, death from drug
followed by cognitive- overdose, HIV infection and
behavioral therapy, crime activity. lt is very safe
psychotherapy and counselling to commence and most
for the patient. So this the commonly used long-term
best advice. programmed for management
of heroin addiction in
Benzodiazepines are Australia.
associated with fatal and non-
fatal opioid overdose among 71. A 49-year-old male,
heroin users. There is high a commercial truck driver by
risk of benzodiazepine profession, presented with
dependence especially in drug left sided chest pain radiating
abuser and a prescription to the both arms and jaw.
longer than 5 days is Electrocardiography shows ST
discouraged. A patient with segment elevation consistent
acute withdrawal symptoms with STEM. Thrombolysis is
can still have 5 days of done with tenecteplase which
benzodiazepine from a GP if resulted in resolution of
GP finds withdrawal symptoms symptoms.
at clinic.
What will you advise him
Telling the patient that about driving?
diazepam is not used for
management of heroin a. He can drive private car
withdrawal symptoms is after 4 weeks
incorrect as benzodiazepines b. He can drive his
such as diazepam may be commercial truck after 4
recommended for opiate weeks
withdrawal especially if there c. No driving restriction
is co-morbid alcohol d. Notify to the police
withdrawal. e. He should drive carefully
Benzodiazepines may and avoid driving on main
temporarily ease the anxiety, highways
muscle spasms, and insomnia
associated with opioid Answer detail:
withdrawal. However this is The correct answer is b.
preferably used in acute
withdrawal while patient is in After acute myocardial
drug rehabilitation program. infarction, the driving
restriction applies as follows:
A severe withdrawal reaction
is sometime precipitated if the -Private vehicles-2 weeks.
patient has recently used -Commercial vehicles-4
heroin or another opioid. It is weeks.
recommended that naltrexone
should not be commenced After 4 weeks, this person can
until seven to 10 days after start driving if remains well
the last use of heroin. otherwise.

Methadone maintenance is an 72. A 42-year-old woman comes


evidence based programmer to your clinic discuss the
report of her recent Pap and varicella) as he has an
smear. The report shows that egg allergy. What is the next
she has low grade squamous step in management?
intraepithelial lesion. She has
no history of negative pap a. Give MMRV on next visit
smears as she was in b. Do not give the vaccine
overseas and she did not see c. Give vaccine in hospital
a doctor there. d. Do an allergy tests
e. Explain her that MMRV
What is the most appropriate does not contain egg protein
next step in her
management? Answer detail:
The correct answer is e.
a. Repeat Pap smear in 12
months The measles, mumps, rubella,
b. Repeat Pap smear in 6 varicella (MMRV) vaccine is a
months combination vaccine that
c. Continue 2 yearly Pap protects against four
smears childhood illnesses-measles,
d. Cease further screening mumps, rubella and varicella
e. Give human papilloma (chickenpox).
vaccine
MMRV vaccine is as safe as
Answer detail: any other vaccine, and an
The correct answer is b. allergy to eggs should not
delay measles vaccination.
If a woman has no history of MMRV vaccine is prepared
negative Pap smears in from chicken fibroblasts and
preceding 2-3 years, and her not from the eggs. So MIMI/
Pap smear shows low grade vaccine can be given to egg
squamous intraepithelial allergic patients.
lesion, she should have repeat
Pap smear in 6 month or 74. Which of the following would
referral for colposcopy. decrease the risk of breast
cancer in pre-menopausal
All other options (Pap smear women?
in 12 months and 2 yearly) is
incorrect. Human papilloma a. Smoking
vaccine is not helpful if the b. Family history of breast
lesion has already developed. cancer
c. Excess body fat
If a woman has history of d. Decreased fruits and
negative Pap smears in vegetables in the diet
preceding 2-3 years and her e. Avoiding breastfeeding
Pap smear shows low grade
squamous intraepithelial Answer detail:
lesion, a 12 month repeat Pap The correct answer is c.
smear is required.
The association of obesity and
73. A mother of 18-month child excessive body fat is different
does not want her child in premenopausal and post-
immunized for MMRV menopausal women.
(measles, mumps, rubella,
Excessive body fat in
premenopausal women is Suitable regimens for
associated with decreased risk meningococcus prophylaxis is
of breast cancer development. ciprofloxacin 500mg orally as
However, obesity and a single dose for adults and
excessive body fat in post- children over 12 years.
menopausal woman is
associated with increased risk Children under 12 years and
for breast cancer. pregnant females should avoid
ciprofloxacin. They should use
Smoking and family history of ceftriaxone as a single dose
breast cancer increases the injection.
risk of breast cancer. Also,
diet containing fewer fruits Rifampicin 600 mg orally 12
and vegetables and avoiding hourly for 2 days is an
breastfeeding for six months, alternate regimen for
increase the risk of breast prophylaxis for meningococcal
cancer. meningitis.

75. A 10-year-old child was However, it is contraindicated


diagnosed with in severe liver disease,
meningococcal meningitis. pregnancy and alcoholism.
His father has severe chronic
liver disease due to hepatitis 76. A 75 year-6d male diagnosed
C and lives in the same with terminal lung cancer
home. came to you requesting an
insertion of intravenous line
Which of the following would to inject lethal drugs to kill
be the best chemo- himself lives with his son and
prophylaxis for his father? does not want to be a burden
for his family. He has been
a. Rifampicin independent throughout Ns
b. Ciprofloxacin life. What would be the most
c. Azithromycin appropriate step you would
d. Gentamicin consider?
e. Dapsone
a. Put intravenous cannula
Answer detail: and prescribe some heavy
The correct answer is b. doses of midazolam.
b. Refer him to community
Chemoprophylaxis for nurse for intravenous access
meningococcus meningitis is c. Tell him that it is illegal
offered to close contacts of to do so
the diagnosed case of d. Refer him to aged care
meningitis. specialist
e. Prescribe him oral
It aims to eradicate medication for pain relief
asymptomatic carriage in
close contacts so that Answer detail:
susceptible members of the The correct answer is c.
group do not acquire the
organism and get an invasive This patient has been
infection. diagnosed with terminal lung
cancer and he is requesting more about the injuries. Ask
physician-assisted suicide. It patient whether she feels
means provision of means to afraid of her partner.
the patient to kill himself. This
may have some moral Emphasize confidentiality so
justification as being the right that patient feels comfortable
of the individual to self- in sharing more information.
determine about his life.
However it is illegal to do so Accurate documentation is
under criminal law in essential for future record. If
Australia. possible, photographs should
Referring him to community be taken of all patients with
nurse or to aged care visible injuries.
specialist is not required. 1 his
patient should be referred to Allow the patient to tell her
palliative care center for story.
comfort measures. Prescribing
him oral pain relief medication Explain that the patient has
not required as in the question legal rights and violence in the
patient did not come with home is as illegal as violence
pain. in the streets. Patient may
choose to inform police.
77. A 33-year old woman comes
to your clinic for Pap smear. Encourage and respect the
During examination, you patient's individual autonomy,
noticed some bruises and self-determination and self-
injuries on chest and esteem.
abdomen. She claims that
these injuries happened 78. A 34-year-old woman has
because of slipping on a tiled been using oral contraceptive
floor. She lives with her pills for past 3 years. She is
partner. planning to conceive this
year. She drinks regular
What would be the most alcohol.
appropriate next step in the
management? What is the most appropriate
advice?
a. Document about injuries
however do not inform her a. Stop alcohol now
b. Explore more about b. Stop alcohol after
cause of these injuries positive pregnancy test
c. Notify the police c. Stop alcohol in 2nd
d. Suggest her that she trimester
may be a victim of domestic d. There is no need to stop
violence e. Do Pap alcohol
smear and see her in clinic e. Avoid pregnancy for 6
once report is available months after stopping alcohol

Answer detail: Answer detail:


The correct answer is b. The correct answer is a.

In this situation, the best Alcohol consumption during


approach would be to explore pregnancy is a risk factor for
fetal alcohol syndrome, birth This is the right age to start
defects, and low birth weight. human papillomavirus vaccine
Binge drinking is particularly in this case.
harmful to fetal
neurodevelopment. At the age of 12 and 13,
human papilloma vaccine is
The safest option for pregnant given to all school going
women is to abstain from children and it is included in
drinking if they are pregnant, National Immunization
planning a pregnancy or Program (NIP) Schedule and
breastfeeding. is funded by Immunize
Australia Program.
So this patient should stop
drinking alcohol now as she is HPV infection caused by HPV
planning pregnancy to types 6, 11, 16 and 18 is
minimize the risk of alcohol- vaccine preventable.
related complications for the
fetus. Types 16 and 18 can lead to
cervical cancer. Types 6 and
79. A mother brings her 13-year- 11 are associated with genital
old daughter to your office to warts.
discuss the prevention of
genital warts and cervical Human Papilloma Virus
cancer? vaccine is available in
Australia since November
Which of the following is the 2006 and two types of
right information to be given vaccines are available,
to her regarding her GARDASIL and CERVARIX.
daughter?
There is very convincing
a. Human papilloma virus evidence that Human
vaccine can only be given Papilloma vaccine can prevent
when she turns 16 cervical cancer.
b. Human papilloma
vaccine is not available in Regular Pap smears are still
Australia yet essential because the HPV
c. There is no evidence vaccine does not prevent all
that human papilloma vaccine cervical cancers and should be
can be helpful in preventing conducted as routine.
cervical cancer
d. This is the right age for 80. The mother of one of your
her to be given human patients calls you, inquiring
papilloma virus vaccine about the reason for her
e. Regular Pap smears are daughter's consult. The
not essential because the daughter is 18 years old.
HPV vaccine does prevent
all cervical cancers What do you tell her?

Answer detail: a. Tell her that you cannot


The correct answer is d. disclose a patient's
information
b. Tell her the reason for
her daughter's consult
c. Tell her to come in Exposure to tobacco smoking
tomorrow with her daughter during pregnancy can increase
d. Tell her to ask her the risk of health hazards to
daughter the baby antenatal and post-
e. Tell her to make another partum.
appointment for her concerns
Smoking in the environment
Answer detail: increases the risk of
The correct answer is a. development of asthma in the
child.
The confidentiality and privacy
of the clinical record of a Following is the list of things
patient, are very important. which are not helpful in
reducing the risk of asthma
If a girl or boy is elder than 16 after the birth of the baby.
years, they are considered as
independent individuals and -Maternal dietary changes or
their clinical information allergen avoidance in
should not be told to anyone pregnancy, for example,
including the parents and avoidance of house dust mite
spouse. or pet allergens.

If the child is a minor (under -Probiotic dietary


16 years of age), the parents supplements, vitamin A, D or E
are entitled to act on behalf of supplements, or fish oil.
their children.
82. A 35-year-old woman came
81. A 24-year-old woman is seen for your advice regarding
by you on the first antenatal screening. She is diagnosed
visit during her current with familial adenomatous
pregnancy. She has a history polyposis at the age 32.Her
of asthma and is keen to do daughter is 5 year old.
everything which will help to
reduce the risk of asthma in What would you advise for
her child. She and her her daughter?
husband, both smoke for last
five years. a. Sigmoidoscopy every
year at the age of 25
Which of the following has b. Colonoscopy every year
evidence to reduce the at the age of 25
likelihood of asthma in her c. Sigmoidoscopy every 2
baby? years after the age of 35
d. Sigmoidoscopy every
a. Quitting smoking year beginning at 10-12
b. Allergen avoidance years
c. Maternal dietary changes e. Refer the case to senior
d. Fish oil colleague
e. Vitamin A supplements
Answer detail:
Answer detail: The correct answer is d.
The correct answer is a.
People with familial
adenomatous polyposis are at
high risk of developing bowel engaging in sexual behavior in
cancer much earlier than the front of a patient or client.
general population.
Engaging in sexual activity
If there is family history of with a person who is closely
familial adenomatous related to a patient or client
polyposis, the person should under the practitioner's care
begin screening from age 10- may also constitute
15 years with a sigmoidoscopy misconduct.
every year until the age of 35.
84. A 60 year-old male visited
After age 35, continue your clinic for routine medical
screening with sigmoidoscopy checkup. He smokes 45
every 3 years. cigarettes per day, drinks
alcohol 10 standard drinks
83. A 32-year-old female per day and his body mass
presented to your office for index is 32.He wears seat-
advice. She told you that she belt 60 percent of the times
has a sexual relationship with when he drives. He is
another GP at your practice. motivated to follow life style
You also noticed your modification advice. Which of
colleague making sexual the following is the most
remarks about that patient. significant advice for
prevention of cardiovascular
What would you next? disease and premature death
in this patient?
a. Write good notes and
save for future reference a. Quit smoking
b. Contact medical b. Reduce alcohol intake
indemnity insurance for c. Wear seat-belt every
advice time
c. Tell her that you need to d. Reduce weight
call the police e. Regular exercise
d. Inform practice manager
e. Notify Australian health Answer detail:
practitioner regulatory The correct answer is a.
agency
Smoking is strongly
Answer detail: associated with high risk of
The correct answer is e. cardiovascular disease.
Quitting smoking is
It is mandatory to notify considered the most effective
Australian Health Practitioner lifestyle interventions for
Regulatory Agency if you have preventing cardiovascular
reasonable believe that your disease and premature deaths.
colleague is involved in sexual In a patient with coronary
misconduct with a current or heart disease, smoking
past patient. cessation is associated with a
36 percent reduction in the
Sexual misconduct also risk of all-cause mortality.
includes making sexual Wearing a properly adjusted
remarks, touching patients or seat belt reduces the risk of
clients in a sexual way, or mortality by 50 percent.
However this mortality is Answer detail:
related to trauma and severe The correct answer is c.
injuries and is not due to
cardiovascular disease. This patient has erectile
Also advise the patient to dysfunction secondary to
reduce alcohol intake to a depression and smoking.
maximum of two standard Alcohol use is in moderation
drinks per day and have at (less than 7 standard drinks
least two alcohol-free days per day) and is unlikely to
per week. If patient is female, cause erectile dysfunction.
she should take up to one
standard drink per day with Heavy drinking is more than 7
two alcohol-free days per drinks per day and it would be
week. associated with erectile
Blood pressure and body dysfunction on long term.
weight have direct association
with each other. One percent So quitting smoking would be
reduction in weight lowers the best advice. Smoking is
systolic blood pressure by 1 associated with increased
mmHg. The benefits of weight cardiovascular risk and can
reductions include decrease in lead to organic erectile
insulin resistance and dysfunction by causing
improvement in atherosclerosis.
hyperlipidaemia.
Regular exercise increases Amitriptyline is less likely to
high-density lipoprotein cause erectile dysfunction and
cholesterol and reduces body is better than sertraline
weight. This in turn reduce regarding sexual side effects.
insulin resistance and
systemic vascular resistance. Known risk factors associated
Regular exercise has an with erectile dysfunction
independent cardio-protective include depression, diabetes,
activity. dyslipidaemia, hypertension,
neurovascular disease,
85. A 45-year-old male patient obstructive sleep apnoea,
with history of depression, is smoking and medications.
on amitriptyline for last 2
years. He now presented with Recommendations of alcohol
erectly dysfunction. He consumptions:
smokes 20 packs/day and
drinks 4 standard drinks/day. 1- For healthy men and
women, drinking no more than
What is the most appropriate two standard drinks on any
management? day reduces risk of harm from
alcohol-related disease or
a. Change amitriptyline to injury over a lifetime.
sertraline 2- Drinking no more than four
b. Prescribe sildenafil and standard drinks on a single
continue amitriptyline occasion reduces the risk of
c. Quit smoking alcohol-related injury arising
d. Reduce alcohol from that occasion
e. Encourage exercise
86. A 32-year-old woman visited diagnosed with bilateral
you for evaluation. She has breast and ovarian cancer
been sexually active for last before the age of 50.
12 years and takes combined
oral contraceptive pills since What will you advise her?
then. She is non-smoker. She
is slightly obese however she a. Two yearly
does exercise daily for 25 mammography
minutes. b. Two yearly ultrasound
c. Refer to family cancer
Which ONE of the following clinic for risk assessment and
the strongest risk of genetic testing
developing cervical cancer? d. BRCA gene screening
e. Six monthly self-breast
a. Oral contraceptive pills examinations
for more than 5 years
b. Non-smoking Answer detail:
c. Sexual activity The correct answer is c.
d. Mild obesity
e. Age A person's first-degree
relative is a parent, sibling, or
Answer detail: child. A first-degree relative
The correct answer is a. shares about half of their
genes with the person.
This patient has been taking
oral contraceptive pills for A second-degree relative of a
longer than 5 years and this person is an uncle, aunt,
make her high risk for nephew, niece, grandparent,
developing cervical carcinoma. grandchild or half- sibling A
second-degree relative shares
High risk factors for about one-quarter of their
developing cervical carcinoma genes with the person.
include:
This woman has high risk
-Oral contraceptive pills for features for breast cancer due
more than 5 years to strong family history of
-Smoking possible genetic breast and
-Immunosuppression ovarian cancer (being Jewish
-Persistence infection of high in origin, bilateral breast
risk human papilloma virus - cancer in same person in the
infection. family, breast and ovarian
cancer below the age of 50).
Sexual activity is an average
risk factor for developing According to RACGP
cervical carcinoma. Mild guidelines, all patients who
obesity with health life style is are found to be at high-risk
unlikely to make high risk for family history of breast and
developing cervical cancer. ovarian cancer, should be
considered for following
87. A 35-year-old Jewish woman things:
presented to you for breast
cancer screening as one of -Advise referral to a cancer
her paternal aunts has been specialist or family cancer
clinic for risk assessment, for at least 4 weeks according
possible genetic testing and to Australian legislation.
management plan.
After that he can have
-Ongoing surveillance unconditional permission to
strategies may include regular drive. Staying in intensive
clinical breast examination, care for 24 hours is normal
and annual breast imaging after repair of the abdominal
with mammography, MRI or aortic aneurysm for strict
ultrasound. monitoring of vitals. It is not
the reason to withhold or
BRCA gene testing would be cancel his license.
required after thorough
assessment at family cancer Referral to general
clinic in the area. practitioner is not required as
you can advise the patient
So option c is correct. post-operatively before he
goes home. This restriction on
88. 65-year-old male had his driving applies
abdominal aortic aneurysm. immediately after the
He was seen by a vascular discharge while he may see
surgeon who repaired the his GP in a week. So it is the
aneurysm and transferred the treating team's responsibility
patient to intensive care unit to instruct the patient.
for 24-hours.Patient made
very smooth recovery and is 89. All of the following situations
now ready to be discharged. are an indication of sexual
He asks if he can drive his misconduct, by health care
car as before. provider, except?

What will be the next course a. Ridiculing of a patient's


of action? sexual preferences or
orientation
a. He is unfit to drive for 4 b. Commenting about
weeks sexual history that are not
b. He is unfit to drive relevant to the clinical
forever as he has been in issue
intensive care unit c. Requesting details of
c. He can drive after he sexual history or sexual
spends a week at home preferences not relevant to
without any symptoms the clinical issue
d. Refer him to his general d. Conversations about the
practitioner to look into the sexual problems or fantasies
matter e. He is unfit to drive of the doctor
for 6 months post-repair e. Talking about sexual
partners with patient who
Answer detail: came for contraceptive
The correct answer is a. advice

After repair of abdominal Answer detail:


aortic aneurysm, a person is The correct answer is e.
unfit to drive his private car
It is not sexual harassment if Which of the following is the
a medical practitioner most appropriate advice for
discusses with the patient her?
about sexual behavior and
sexual history if she presents a. Refer her for
for contraceptive advice. It is colonoscopy now
very relevant to the clinical b. Suggest colonoscopy
issue they are dealing with. after every two years starting
at the age of 30
According to guidelines c. Refer her to bowel
provided by medical board of surgeon for urgent colectomy
Australia, all of the following d. Start bowel cancer
situations account to sexual screening at the age of 35
misconduct and sexual e. No follow up needed a
harassment: she refused genetic testing

- Making an unsolicited Answer detail:


demand or request, whether The correct answer is a.
directly or by implication, for
sexual favors. Hereditary nonpolyposis
- Irrelevant mentioning of a colorectal carcinoma (HNPCC)
patient's or doctors sexual is an autosomal dominant
practices, problems or disorder responsible for 3 to
orientation. 5% or cases of bowel cancers.
- Ridiculing of a patient's
sexual preferences or If a person has a parent
orientation. (mother or father) with
- Commenting about sexual HNPCC, he/she has a 50/50
history that are not relevant chance of inheriting the
to the clinical issue. mutated gene.
- Requesting details of sexual
history or sexual preferences First degree relatives of
not relevant to the clinical patients with HNPCC should
issue. have colonoscopy every 1-2
- Conversations about the year beginning between the
sexual problems or fantasies age of 20-25 and yearly faucal
of the doctor. occult blood testing (FOBT)
- Making suggestive (according to Cancer Council
comments about a patient's of Victoria).
appearance or body.
As this patient has family
90. A 25-year-old lady comes to history of bowel cancer
your clinic for advice (HNPCC in mother) in 2 first
regarding bowel cancer degree relatives before the
screening. Her mother had age of 50, she should be
hereditary nonpolyposis considered high risk for
colorectal cancer and was hereditary nonpolyposis
treated surgically at 40-year colorectal carcinoma and she
of age. Her grandmother also should be offered genetic
had bowel cancer at the age testing for HNPCC gene.
of 49. She is not interested
to have genetic testing. If she refuses to go for
genetic testing, it should be
assumed that she has HNPCC 92. A 32-year-old woman
gene and should have 2-yearly presented to your clinic at 22
colonoscopy starting from now weeks of gestation. She is
and yearly FOBT to pick up quite upset and reports
bowel cancer at early stage. intimate partner violence for
last few months.
91. When should suspected child
sexual abuse be reported? Which of the following is
likely to increase the risk of
a. When you are sure that intimate partner violence?
child is telling the truth
b. Only when you have a. Non-pregnant women
confirmed gonorrhea infection b. Less frequent
c. After you have contacted attendance to general
the parents practitioner clinic
d. Only when you have c. Treated mental illness
proof of the abuse d. Chronic unexplained
e. When you have a belief physical symptoms
based on reasonable grounds e. Past physical trauma
that child sexual abuse
has occurred or is occurring Answer detail:
The correct answer is d.
Answer detail:
The correct answer is e. The risk of intimate partner
violence should be considered
A doctor or nurse is legality' in following high risk
requited to report the population:
suspected child sexual abuse
to child protection services, - Women with symptoms of
when there is a belief based mental illness.
on reasonable grounds that
child sexual abuse has - Women with chronic
occurred or is occurring. unexplained physical
symptoms.
You do not need to be sure
whether child is telling the - Pregnant women.
truth or not. You do not need
to contact the parents before - Frequent GP visitors.
reporting to child protection
services. No proof of sexual -Untreated mental illness.
abuse is required for
mandatory reporting of child - Past psychological trauma.
sexual abuse.
Non-pregnant women are less
When you suspect a child likely to be at risk of intimate
sexw31 abuse, immediately partner violence as compared
report to the local child to pregnant women.
protection services and they
investigate the matter. There 93. A 32-year-old male from
is no penalty if no child sexual Sydney comes to you clinic
abuse is confirmed and you for assessment. He reports 5-
reported it in a good faith. day history of penile
discharge. He denied having
any fever, rash or any other
symptoms. Definitive diagnosis of N.
gonorrhoea infection can be
He is sexually active with made with nucleic-acid
several female partners, with amplification testing of a
inconsistent condom use. urethral or urine sample; this
test has largely replaced
The patient's vital signs are culture in clinical practice. If
normal. He has normal male available, Gram's stain of the
external genitalia without purulent exudate typically
inguinal lymphadenopathy or shows gram-negative
lesions; purulent material is diplococci.
expressed from the urethral
meatus. Recommended therapy for
gonococcal urethritis consists
PCR testing of the swab is of a single dose of ceftriaxone
positive for Neisseria 250 mg intramuscularly plus a
gonorrhoeae and negative for single dose of azithromycin 1
Chlamydia trachomatis gram by mouth.
infection.
Ceftriaxone alone is not
Which one of the following recommended because of high
therapies is most appropriate rates of concomitant infection
for this patient? with Chlamydia, which is
treated with either
a. Single doses of azithromycin or doxycycline.
intramuscular ceftriaxone and
oral azithromycin In addition, there have been
b. Single dose of recent reports of ceftriaxone-
intramuscular ceftriaxone resistant gonorrhea, and the
c. Single dose of second drug may provide
intramuscular benzathine additional coverage.
penicillin G
d. Oral ciprofloxacin for 5 Oral fluoroquinolones (such as
days ciprofloxacin) and oral
e. Single dose of oral cephalosporins (such as
cephalexin cephalexin ) are no longer
recommended in this setting
Answer detail: because of concerns about
The correct answer is a. resistance.

The most appropriate therapy N. gonorrhea may be resistant


for gonococcal urethritis is a to penicillin, which is the drug
single dose of ceftriaxone 250 of choice for syphilis infection
mg intramuscularly plus a
single dose of azithromycin 1 94. A 32-year-old lady presented
gram orally. The most likely to your office for advice
causes of uncomplicated, regarding screening. Her
purulent urethritis in a grandfather was diagnosed
sexually active young man are with colon cancer at the age
Neisseria gonorrhoeae and of 76. Her elder sister had
Chlamydia trachomatis cholecystectomy at the age of
infection. 46. Her brother was
diagnosed with diabetes including acute myocardial
mellitus at 56 year of age. infarction and stroke.
2- All the women with a
What would be your advice history of gestational diabetes
for her? mellitus.
3- Women with polycystic
a. Colonoscopy every 12 ovary syndrome.
months 4- Those on antipsychotic
b. Oral glucose tolerance drugs.
tests every 12 months 5- Those with impaired
c. Serum cholesterol every glucose tolerance test.
12 months
d. No need for screening This woman does not fall in
now this high risk category and so
e. Request genetic risk does not need oral glucose
assessment tolerance test.

Answer detail: 95. A 74-year-old male presented


The correct answer is d. with inability to speak, facial
weakness and right sided
This patient is slightly above paralysis. CT scan shows
the average risk of developing ischemic stroke involving
colon cancer due to positive more than 65% left
family history of colon cancer hemisphere. His wife do not
with one second degree want any active invasive
relative (grandfather). management. However his
son tells you to do
She should have FOBT at least everything.
every two years after the age
of 50. Consider sigmoidoscopy What will you do in this
every 5 years from the age of situation?
50.
So this woman does not need a. Tell the wife that it is
any colonoscopy at this stage. too soon to decide on
discontinuing the life support
All adults aged 45 and over b. Do the treatment as it is
should have 5 yearly check of an emergency
serum cholesterol for c. Call guardianship court
screening of d. Give intravenous
hypercholesterolemia. nutritional support
e. Give nasogastric fluids
Fasting blood glucose should and noninvasive treatment
be done every 3 years in all
patients above 40 years of Answer detail:
age. The correct answer is e.

Fasting blood glucose every 3 This patient has big ischemic


years, should be considered at stroke and need urgent
any age in following high risk treatment and intensive
groups. monitoring for possible
hemicraniotomy. This
1- All people with a history of situation is a contraindication
previous cardiovascular event
to thrombolysis due to risk of Which of the following is the
hemorrhagic transformation. next step in management?

The consent should be a. VDRL reaction


obtained from following b. Gram stain and culture
people where the patient is of penile discharge
incapable of consenting to c. Gonococcal complement
medical treatment and has not fixation test
had a guardian appointed. d. Prostatic massage
e. Dark ground illumination
1- A guardian of the person of the discharge
needing the treatment.
Answer detail:
2- The spouse or de facto The correct answer is b.
partner of the person needing
the treatment. The patient's presentation is
suggestive of an acute
3- A person who, on a regular gonococcal urethritis.
basis, provides or arranges for
domestic services and support In men, about 50% acute
to the person needing the infections are
treatment but does not asymptomatic.The symptoms
receive remuneration for of gonococcal urethritis
doing so. include a penile discharge and
dysuria. The discharge is often
4- A person who is the nearest present spontaneously at the
relative (other than the urethral meatus, purulent or
spouse or de facto partner) of mucopurulent in color, and is
the person needing. copious in amount.

5- Any other person who Risk factors for gonorrhoea


maintains a close personal include:
relationship with the person
needing treatment. - New sexual partner or
multiple sexual partners.
6- A person prescribed in the - Being unmarried.
regulations. - Young age.
- Low educational and
So in this situation, the socioeconomic levels.
patient himself cannot consent - History of substance abuse.
to the treatment due to stroke Previous episode of
and his spouse would be the gonorrhoea.
person who can make decision
about provision of health care. The gonococcal organism is
So option e is correct rapidly identified by Gram
response. stain and culture of penile
discharges.
96. An 18-year old male escort
comes to your office due to Due to its high sensitivity and
episodes of dysuria. He also specificity, a Gram stain of a
noted yellowish penile male urethral specimen, which
discharge. demonstrates
polymorphonuclear leukocytes
with intracellular Gram- An intervention is conducted
negative diplococci, can be on a randomly selected group
considered diagnostic for of people and compared with a
infection with Neisseria matched control group not
gonorrhoea in men. subject to the specific
intervention. The objective is
VDRL is used for the diagnosis to establish a causal
of syphilis. Prostatic massage relationship between the
is an unreliable method of intervention and the
obtaining a test sample for hypothesized outcome.
acute gonorrhoea.
Cohort (or prospective) study
Dark ground illumination is a is also referred to as 'follow-
method for demonstrating the up'. The study follows a group
presence of Treponema (cohort) of individuals with a
pallidurn. Antibiotic specified characteristic or
management of acute disease over a period.
gonorrhoea is with ceftriaxone Comparisons are made with a
250 mg IM as a single dose. control group.
Re-culture the organism in 3-
I days after treatment. Cross-sectional or prevalence
Contact tracing is essential. study follows a correlation
approach using existing
97. You are working as a GP in a databases. It is a survey of
rural area. You are keen to the frequency of disease, risk
do some research in your factors or other
area. You decided to do a characteristics in a defined
research study the clinical population at one particular
effects of aspirin on time.
osteoarthritis.
A case study is a research
Which study is most effective method involving an up-close,
study design? in-depth, and detailed
examination of a subject of
a. Cohort study study (the case), as well as its
b. Cross-sectional study related contextual conditions.
c. Case study
d. Case-control study Case-control (or
e. Randomized controlled retrospective) study is an
trials observational study in which
people with a disease (cases)
Answer detail: are compared with those
The correct answer is e. without it (control group).

Randomized controlled trials 98. A 30-year-old woman is


are the most rigorous way of concerned about breast
determining whether a cause- cancer has her mother was
effect relation exists between diagnosed with breast cancer
treatment and outcome and at the age of 48. Clinical
for assessing the cost examination of the breast is
effectiveness of a treatment. normal.
What is the most appropriate The correct answer is e.
advice for her?
All the partially immunized
a. Mammography every and un-immunized adults
year from the age of 40 should receive active
b. Mammography every 2 immunization with tetanus
years from age of 50-69 toxoid.
c. Refer to surgeon
d. Refer for genetic studies The primary series for adults
e. Reassure her that she is consists of three doses. First
not at high risk of breast and second dose are given 4
cancer weeks apart and third dose is
given by 6 to 12 months.
Answer detail:
The correct answer is a. A clean wound with clear
history of tetanus status more
If a woman has a first degree than 3 doses does not require
relative less than 50 years of both tetanus toxoid and
age diagnosed with breast tetanus immunoglobulins. So e
cancer, she should have is correct response.
mammography every year
from the age of 40. Intravenous and oral
antibiotics are usually
All other options are incorrect. reserved for dirty wounds and
cellulitis. However clean
99. You are working in a rural wound does not need
hospital emergency antibiotics at all.
department. Your next
patient is a 22-year-old male 100. One of your patients,
who had minor trauma to his meets you on the street mall
left leg while he fell from his and invites you for a coffee
motorbike this morning. His and a chat, and may be lunch
wound looks clean and he later.
had 3 doses of tetanus toxoid
about 3 years ago. She just had seen you at
your clinic two days ago.
What is the most appropriate
management of this patient? What is true about
socialization between doctors
a. Tetanus toxoid and patients?
b. Tetanus
immunoglobulins 250 units a. You can go out for a
intramuscularly coffee but not more than that
c. Tetanus toxoid and b. You can establish a
tetanus immunoglobulins contract to finalize her
d. Give intravenous psychotherapy and
antibiotics and tetanus afterwards, you can socialize
immunoglobulins c. You can socialize as you
e. Neither passive nor are both consenting adults
active immunization is d. You should not socialize
required at this stage with your patients under any
circumstances
Answer detail:
e. Ask patient to give the Patient is keen to know for
phone number so that you how much time he is not
can talk latter allowed to drive?

Answer detail: a. 4 weeks


The correct answer is d. b. 3 months
c. 6 months
Doctor patient relationship d. 12 months
has professional limitations on e. 3 years
doctor not to get involved in
any relationship with a Answer detail:
patient. The correct answer is b.

- Socializing with patients or When a person develops


with previous patients is seizure while being well
discouraged and considered controlled on medication
unethical. without seizure for 12 months
and no cause of seizure is
- Having a coffee together identified, patient should not
further increases the risk of drive for 3 months.
socializing beyond that.
After 3 months if he remains
- A physician must not seizure-free, he may be
socialize with patients under eligible for conditional driving
any circumstances. license after medical review.
Compliance to medication and
- Any romantic or sexual good seizure control are key
relationship which results requirements for driving
from socializing with the license in a patient with
patients is also unethical. epilepsy.

- Asking for phone number in If a provoking factor (sleep


a street is also inappropriate. deprivation, electrolyte
abnormality, alcohol abuse) is
If you need to contact her found which lead to seizure
regarding her health, you can activity and if it can be
use hospital record to get her avoided, then for 4 weeks,
contact details. patient should not drive.
After finishing 4 weeks,
101. A 45-year-old male patient should be considered
presented with a self-limiting for conditional driving license
episode of seizure in after medical review
emergency department. He is
known to have epilepsy which The above information is also
is well controlled with available on
carbamazepine for last 12 www.austroads.com.au.
months. Blood tests do not
show any abnormality and 102. A 42-year-old woman came
there is no obvious cause of to your clinic for advice. She
the seizure identified. is a secretary in an office and
she smokes. She does self-
breast examination every
month and clinical
examination of her breast by b. Ultrasound
her general practitioner every c. Weekly self-breast
2 years. She also does some examination
physical activity at work and d. Refer for screening
during leisure time. mammography every year
e. Reassure
Which ONE of the following is
will lower the risk of breast Answer detail:
cancer? The correct answer is e.

a. Smoking Annual breast cancer


b. Self-breast examination screening is recommended for
c. Physical activity at work women aged 40 years or over
d. Clinical breast who have one or more of the
examination following criteria:
e. All of the above
-A first-degree relative
Answer detail: (mother/sister/daughter,
The correct answer is c. father/brother/son) with
breast cancer diagnosed
There has been a consistent before the age of 50.
evidence that physical activity -A first-degree relative with
at work and during leisure cancer in both breasts
time reduces the risk of breast (diagnosed at any age).
cancer by 20-40% in women -Two or more first-degree
of both reproductive and non- relatives with breast cancer
reproductive age. (diagnosed at any age).

Self-breast examination and So this patient does not need


regular clinical examination of mammogram now and needs
the breast is no longer is reassurance.
recommended as evidence
suggest that these tools do For general population, breast
not decrease the number cancer screening is aimed at
deaths from breast cancers. well women without
symptoms, aged between 50
Smoking increases the risk of and 69 years (women aged
developing breast cancer. 40-49 and 70 years and older
are also able to attend if they
103. A 37-year-old woman wish).
comes to your clinic with
concerns about breast Ultrasound is only indicated if
cancer. Her mother was patient develops lump in the
diagnosed with breast cancer breast and needs further
at the age of 60 years. She is evaluation before the age of
asymptomatic and her clinical 40. It is not helpful in the
examination is normal. presence of normal clinical
examination.
What is the most appropriate
next step in her Mammography is not useful
management? before the age of 40 due to
dense breast tissue.
a. Mammography
104. A female psychologist is - Socializing with the patient.
treating a 25-year-old male - Calling each other by first
patient for complicated grief name.
related to the death of his - Treating the patient as a
parents in an automobile friend and touching or
crash. The psychologist lost frequent hugs.
her own parents when she
was a university student. She Abandonment of the
identifies herself strongly therapeutic alliance describes
with this patient. The a situation in which clinician
psychologist often sees him may pay less attention to a
after regular clinical hours patient. This may encourage
and usually cancels her other the patient to leave treatment.
appointments in order to see
him on a short notice. Confidentiality breaches occur
when clinicians share
Which one of the following information about patients
ethical pitfall is this with others inappropriately.
psychologist in danger of
committing? If a doctor works for any
federal agency, confidentiality
a. Breach of confidentiality cannot be warranted. This is
b. Divided loyalties as a an example of divided
dual agent loyalties as a dual agent.
c. Erosion of professional
boundaries 105. Which one of the following
d. Practicing outside his malignancies carries the
scope of clinical competence worst prognosis?
e. Abandonment of the
therapeutic alliance a. Metastatic
choriocarcinoma
Answer detail: b. Metastatic seminoma
The correct answer is c. c. Hodgkin lymphoma
d. Non-Hodgkin lymphoma
This clinical scenario involves e. Metastatic breast
a patient who can be carcinoma
considered difficult because
he resembles the clinician in a Answer detail:
way that threatens her ability The correct answer is e.
to maintain professional
boundaries. Breast cancer is the most
common cancer in women in
Potential non-sexual boundary Australia and the second most
violations include: common cancer to cause death
in women, after lung cancer.
- Excessive self-disclosure.
- Special fee arrangements. As many as 40% of women
- Extending time beyond what with recurrent or metastatic
was initially agreed. breast cancer survive at least
- Allowing telephone calls five years. The median
between sessions. survival for patients with
- Extra-therapeutic business metastatic (stage IV) breast
relationships. cancer is 18 to 24 months.
Australia have a five year
Amongst the all given options, survival rate of 71%.
metastatic breast cancer
carries the worst prognosis. 106. A 45-year-old woman
presented for assessment.
Seminoma of the testis is She is concerned about her
associated with a good risk of developing ovarian
prognosis, with most tumours cancer. She has been using
following an indolent course. oral contraceptive pills until 2
In cases without evidence of years ago when she was
lymphatic spread, 5 year diagnosed with BRCA-1
survival approaches 100%. positive breast cancer. Her
Even in the presence of early mother was diagnosed with
lymphatic spread, five year ovarian cancer at the age of
survival is approximately 65.
95%.
Which ONE of the following
Choriocarcinoma is a highly carries the lowest risk for
malignant germ cell tumour developing ovarian cancer?
which usually follows an
abnormal pregnancy with a a. Oral contraceptive pills
hydatidiform mole. It may also b. Age more than 40
occur after a spontaneous c. BRCA-1 positive breast
abortion. cancer
The tumour metastasizes d. Family history of ovarian
early, by means of vascular cancer
invasion and blood spread. e. None of the above
When combined, surgery and
cytotoxic chemotherapy, lead Answer detail:
to improved survival prognosis The correct answer is a.
more than 80% survival in 5
years. Previous use of oral
contraceptive pills carry a
Lymphomas are the most very small risk of developing
common form of ovarian cancer. So this is the
haematological or blood correct
cancer in Australia, and the Answer.
sixth most common form of
cancer overall. There are two Age more than 40 is not a risk
main types of lymphoma-non- factor for developing ovarian
Hodgkin lymphoma and cancer.
Hodgkin lymphoma. More than
90% of lymphomas are non- Family history of ovarian
Hodgkin. cancer and personal history of
SPCA-1 positive breast cancer-
For people diagnosed with both of these put this woman
Hodgkin lymphoma, prognosis at very high risk for
is generally good, with a five developing ovarian cancer.
year survival rate of more
than 87% in Australia. 107. A 38-year-old male
presented to your office for
People diagnosed with non- evaluation. He has fair skin
Hodgkin lymphoma in color and no history of
sunburns as a child. He has
tendency to tan rather than 108. A-14-year-old girl, who has
burn when exposed to left the house and is living in
sunlight. Past medical history a shelter, came to you for
include excision of a skin certain treatment. While
lesion from the right leg having consultation, she says
which was squamous cell that she is fed up of living in
carcinoma with clear edges. the shelter. But she left
house because of domestic
Which ONE of the following violence. You also notice that
the highest risk factor for she has numerous bruises
developing another squamous and scratches on her face and
cell carcinoma? hands.

a. Fair skin color What will be the appropriate


b. Tendency to tan rather next step?
than burn
c. Absence of sunburns as a. Ask her to go back to
a child friends/parents' house
d. Past history of b. Ask her to visit your
squamous cell carcinoma clinic every day
e. Age c. Inform the police
d. Give the number of child
Answer detail: protection agency
The correct answer is d. e. Visit her shelter

Following is list of factors Answer detail:


which put a person at the The correct answer is c.
highest risk of developing
squamous cell carcinoma of Both child protection and
the skin: domestic violence mandatory
reporting requirements vary
-Age over 40 considerably among
-Past history of non-melanoma Australia's states and
skin cancers territories.
-Tendency to burn rather than
tan when exposed to sun Under mandatory child
protection reporting
Fair complexion is a risk factor requirements, a range of
for developing squamous cell professionals, such as health
carcinoma of the skin however professionals, teachers and
it is not useful in assessing welfare workers are required
the severity of the risk factors to report to the police or child
contributing to the disease. protection authorities any
child they suspect is being
So this patient is younger than abused.
40, has tendency to tan rather
than burn. So these are not Most Australian jurisdictions
correct options. Past history do not have mandatory
of squamous cell carcinoma of reporting requirements for
the skin is a big risk factor for domestic violence.
developing another similar
lesion.
Under the Northern Territory's
Domestic and Family Violence -Influenza
Act, however, any adult must -Yellow fever
report domestic violence to -Q fever
the police if they reasonably
believe that a person has or is All patients who have an
likely to suffer serious allergy to egg but not the
physical harm. On receiving a anaphylactic reaction can be
report of domestic violence, a given these vaccines as a
police officer must take single dose.
reasonable steps to ensure
that the report is investigated MMR (measles, mumps, and
rubella) vaccine can be given
This girl is likely to suffer safely to children who have an
harm or violence and police egg allergy. This vaccine is
must be notified immediately made from chicken fibroblasts
to help her and ensure her and not from egg proteins.
safety. Police will further
involve child protection 110. A 45-year-old father was
services. recently diagnosed with
Giving her contact number for tuberous sclerosis. His wife
child protection services is not does not have the gene for
appropriate as this will not this disease.
ensure safety of this domestic
violence victim. What is expected of his
children in relation to
109. A 12-month old male child inheriting the disease?
is brought to your office by
his mother. He has a history a. Half normal, half
of anaphylactic reaction to affected
egg 6 months ago. b. Half of sons affected,
half of daughters carrier
Which of the vaccines he c. None affected, half
would need medical carrier
supervision in hospital d. All carriers, none
settings and in split doses? affected
e. Variable inheritance
a. Measles, mumps, rubella
vaccine Answer detail:
b. Influenza vaccine The correct answer is a.
c. Inactivated polio vaccine
d. Pneumococcal vaccine Tuberous sclerosis is an
e. Hepatitis A vaccine autosomal dominant disorder
therefore half of the offspring
Answer detail: will be affected, and half will
The correct answer is b. be normal.

All patients who have an It is due to mutation in


anaphylactic reaction to the chromosomes 9 and 16. The
egg should receive following triad includes facial rash,
vaccines in split doses under intellectual disability and
medical supervision in seizures.
hospital settings.
111. A 60-year old male and naltrexone should not be
presented for your opinion. prescribed together.
He is alcohol dependent and
is very keen to get treated. Naltrexone is contraindicated
You plan to administer in:
naltrexone.
1. Patients receiving opioid
Which one of the following analgesics.
statement is true regarding 2. Patients currently
naltrexone? dependent on opioids since an
acute withdrawal syndrome
a. It is given for chronic may ensue.
marijuana users 3. Patients in acute opioid
b. It is ideal for withdrawal.
intravenous drug user since it 4. Any individual with acute
is long acting hepatitis or liver failure.
c. It is a substitute for
methadone 112. A 25-year-old Australian
d. It is a generic name for woman, up to date with her
naloxone immunizations, now
e. It is short acting presented to your office for
medical advice. She is
Answer detail: planning to travel to India in
The correct answer is c. next few weeks.

Naltrexone is essentially a What vaccination you should


pure opiate antagonist. consider for her?
Naltrexone is a long-acting
opioid antagonist which is a. Pneumococcal
well-absorbed orally. b. Meningococcal
A severe withdrawal reaction c. Tetanus toxoid and
may be precipitated if the diphtheria
patient has recently taken d. Typhoid
heroin or another opioid. e. Cholera

It is recommends that Answer detail:


naltrexone is not commenced The correct answer is c.
until 7 to 10 days after the
last use of heroin. If heroin or In Australia, until the age of
another opioid is taken by a 13, she must have received
patient who has already been vaccines against polio,
taking naltrexone, all opioid hepatitis 8, measles, mumps,
effects are blocked. rubella, diphtheria, pertussis,
It is not used in management tetanus, H.influenza and
of marijuana users. human papilloma virus.
It is mainly used in treatment
of alcohol dependence and However, the protection
maintenance of opioids against tetanus requires an
abstinence. Naltrexone is a initial course of 3 injections
substitute of methadone in followed by a booster every
maintenance of opioid ten years. So she is due to
abstinence. Both methadone have tetanus toxoid booster.
This applies to all travelers to sepsis from non-healing
all countries. dermatitis.

Cholera vaccine is not So now it is advised to protect


officially recommended by a child from abuse by the
WHO because it has only beliefs of their parents that
limited effectiveness. Vaccine could potentially be harmful to
against typhoid is can be the child. It is moral and
considered while travelling to ethical obligation to make a
India due to poor standards of formal report to the local child
sanitation if patient protection services.
volunteers.
A court order to treat the child
Pneumococcal vaccine should may be granted after
only be considered only in intervention by child
high-risk individuals. protection services however it
is not the next best step.
113. A 6-year-old child is
brought for assessment of his 114. A 33-year-old woman is
skin problems by his parents. visiting you in your primary
You suspect atopic dermatitis care office as a new patient.
as a possible diagnosis and She is healthy otherwise. Her
advise topical corticosteroids. mother was diagnosed with
Parents of the child are not colorectal cancer at the age
willing to use steroids for of 50.
their child.
When should this patient be
What will you do next? screened for colorectal
cancer?
a. Request a court order
b. Notify police a. At the age of 40
c. Notify child protection b. At the age of 50
services c. At the age of 60
d. Cancel the steroid d. At the age 30
prescription e. There is no proven
e. Refer for second opinion benefit for colorectal cancer
screening
Answer detail:
The correct answer is c. Answer detail:
The correct answer is a.
The gold standard for
treatment of atopic dermatitis Screening for bowel cancer
is topical corticosteroids. So should begin approximately 10
cancelling the prescription is years before the age of
not appropriate. diagnosis of colorectal cancer
in a first-degree relatives like
There has been a court case in parent or sibling.
Australia where a child was
diagnosed with atopic Given that this patient's
dermatitis and parents did not mother was diagnosed at the
use the steroids prescribed by age of 50, this patient should
their general practitioner. The have screening at age 40.
child died due to secondary
The natural history of a colon Alcohol is responsible for 30%
polyp to develop into cancer is of road accidents, 40% of fire
thought to be 10 years. injuries, 34% of falls and
drowning, 16% of child abuse
Colorectal cancer screening cases and 12 percent of
has proven mortality benefit suicides.
and it is wrong to say that it
has no proven benefit. Alcoholic liver cirrhosis is the
most common cause of death
115. A 47-year-old male amongst those who survive
presented for his regular through acute intoxication.
health checkup. He is
normotensive and is not over More people die from
weight. He plays golf accidents related to alcohol
occasionally on weekends and and relatively less number of
drinks alcohol 70 grams per people suffer from cirrhosis
day. and die each year in Australia.
So d is correct response here.
What would be the most
likely cause of death in next Other causes of death
10 years? secondary to alcohol abuse
include stroke, ischemic heart
a. Stroke disease, falls and suicide.
b. Cirrhosis of liver
c. Ischemic heart disease 116. You are working as a GP in
d. Accident a rural area and your next
e. Suicide patient is chronic abuser of
marijuana.
Answer detail:
The correct answer is d. Which of the following is
correct regarding chronic
A standard alcohol drink marijuana abuse?
contains 10 grams of alcohol.
a. Incidence of COPD
A male can drink up to 2 decreases
standard drinks per day and it b. Masks underlying
reduces risk of alcohol related psychosis
injuries significantly. This c. Reduces anxiety
patient is drinking 70 g/day d. Impairs the ability to
which makes it 7 standard drive car
drinks daily. e. Increases job
This is heavy alcohol abuse performance
and can lead to short term as
well as long term problems. Answer detail:
The correct answer is d.
Acute alcohol intoxication is
the most common cause of Chronic marijuana abuse
alcohol-related problems, causes following main issues:
leading to injuries and
premature deaths. It accounts - Laryngitis
for two-thirds of the years of - Chronic obstructive
life lost from drinking. pulmonary disease.
- Unmasks underlying
psychosis and increases If an elderly person has no
anxiety. capacity, then guardian or
- Impairs the ability to drive a caretaker should be contacted
car. for consent. After consenting,
- Decreases performance at a social worker should be
job. involved to organize a family
meeting to assess the
117. An 82-year-old man situation.
presented with multiple
bruises on various body So In this situation you should
parts. He performed serial-7 follow up this patient to
test very efficiently. He lives review for complete healing of
with his daughter and tells bruises and possible
you that his daughter's monitoring of the situation.
boyfriend abused him
physically. He strictly forbids He has done serial-7 test
you from informing the police which showed he is having
as he believes that he could good cognition and capacity to
tackle the situation by decide. However, if he
himself at home. declines to consent for
reporting to the police, it must
What would be your approach be respected.
in this case?
118. Which ONE of the following
a. After management of is the most common
bruises let him go to home preventable cause of non-
b. Arrange regular follow melanoma skin cancers?
up but keep his secret
c. Ask him Li see the social a. Exposure to arsenic
worker before he leaves b. Exposure to ultraviolet
hospital radiation
d. Inform the daughter c. Heart transplant
about abuse so that she d. Renal transplant
knows what is going on e. Family history of skin
e. Notify the police cancer
immediately
Answer detail:
Answer detail: The correct answer is b.
The correct answer is b.
In Australia, the most common
If an elderly man is found to preventable cause of non-
have been abused physically, melanoma skin cancer is
financially, sexually or ultraviolet radiation exposure.
neglected intentionally or
unintentionally, it is your Other options (exposure to
responsibility to help him out. arsenic, immunosuppression
post-heart and renal
If he can decide, informed transplant, family history of
consent is an essential step to skin cancer) increase the risk
proceed. The elderly person's of developing non-melanoma
right to decline assistance skin cancer however exposure
should be respected. to ultraviolet radiation is the
most common preventable not require both tetanus
cause of these cancers. toxoid and tetanus
immunoglobulins.
119. You are working in a rural
hospital emergency Intravenous and oral
department. Your next antibiotics are usually
patient is a 20-year-old male reserved for dirty wounds and
who had minor trauma to his cellulitis.
left leg while he fell from his
motorbike this morning. However clean wound does
not need antibiotics at all.
His wound looks clean
however he does not 120. A 13-year-old girl comes to
remember clearly about your clinic for an advice on
status of tetanus vaccination. contraception. She informs
you that she is in a
What is most appropriate relationship with her
management for him? boyfriend and they are
planning to go on a trip out
a. Tetanus Toxoid of the state.
b. Tetanus
immunoglobulins 250 units What is the best next step?
intramuscularly
c. Tetanus toxoid and a. Immediately call her
tetanus immunoglobulins father and ask him to consent
d. Give intravenous for his minor daughter
antibiotics and tetanus b. Give the prescription
immunoglobulins and educate about prevention
e. Give oral antibiotics and of sexually transmitted
tetanous toxoid infections
c. Make an appointment
Answer detail: next week to see her and her
The correct answer is a. boyfriend at your clinic
d. Contact the
All the partially immunised guardianship board and get a
and un-immunised adults consent over phone
(also those who cannot e. Advise her not to go out
remember their immunisation with her boyfriend
status) should receive active
immunisation with tetanus Answer detail:
toxoid. The correct answer is b.

The primary series for adults In Australia, a medical


consists of three doses. practitioner is required to get
consent from parents about
First and second dose are treating minor (less than 16
given four weeks apart and years).
third dose is given by 6 to 12
months. However when writing a
prescription of contraception,
A clean wound with clear it is required to prevent
history of tetanus vaccination unwanted pregnancy and
status more than 3 doses does sexually transmitted
infections by use of condoms. - Unacceptable behavior of the
So the best way of doing this patient like verbal abuse,
is to give a prescription of threatened or actual violence
contraceptives and educate and harassment.
her how she can prevent
unwanted pregnancy and This patient has violated the
sexually transmitted boundary where it is hard to
infections. continue health care in
professional manners. So
All other options are not according to guidelines by
correct in Australian health Australian Medical
system. Association, in such situations
doctor-patient relationship
121. You are working in a GP should be terminated however
clinic. One of your female with due respect for the
patients invites you for a patient.
dinner outside with clearly
romantic intentions. You feel Patient should be notified and
she is attracted towards you. offered to transfer a copy of
her medical record to new
How will you proceed? health care provider.

a. Accept the offer and Ending the doctor-patient


plan a meeting outside relationship should not be
b. Decline the request and misinterpreted by the patient
continue doctor-patient as an act of discrimination
relationship c. Decline the based on gender, sexual
request and discontinue preference, race, marital
doctor-patient status, disability, age and HIV
relationship status.
d. Tell her that its
inappropriate meet her Also you should continue care
outside but still I will if the patient is suffering an
manage to come acute illness. In such situation
e. Notify the management the patient is entitled to have
for in-appropriate behavior of appropriate continuity of care
the patient but still and doctor-patient
continue doctor-patient relationship should not be
relationship terminated.

Answer detail: 122. A 75-year-old male is


The correct answer is c. brought to the hospital with
reduced level of
There are certain situations consciousness. CT scan of the
when a doctor-patient head showed epidural
relationship can be haematoma which requires
terminated. non-urgent surgery. You are
These include: unable to discuss the
procedure with the patient
- Professional boundary due to decreased level of
violations by the patient or consciousness. His spouse
doctor. and the eldest son are
present at the scene. His
spouse says that her husband while considering the patient
never wanted to come to the incompetent to make a
hospital and would never decision.
want to be resuscitated or
any surgery on him. His son Application for Guardianship is
requests you to do everything required for all those patients
possible to save his father's who lack the capacity to
life. decide about their treatment
however it usually takes few
How will you proceed in this days to process.
situation?
Under the Guardianship Act
a. Listen to his son 1987, a 'person responsible'
b. Arrange a family can make decisions about
meeting to reach the most medical or dental
consensus treatments.
c. Apply for guardianship
d. Listen to his spouse A 'person responsible' is, in
e. Request the medical order of priority:
suprintendent to assess the
patient capacity 1. The legally appointed
guardian of the person
Answer detail: (including enduring guardian)
The correct answer is d. with the function of
consenting to medical/dental
If a person cannot give treatment. Or if there is no
consent for their own guardian
treatment and there is no 2.Their spouse or de facto
advance health directive spouse or same sex partner,
present, a health practitioner or if there is no spouse or de
should obtain consent from facto spouse or same sex
the "person responsible” in partner.
this situation, patient’ spouse 3. Their unpaid career or, if
can consent for the patient there is no career.
future treatment. A spouse 4. Their relative or friend who
opinion is considered more has a close relationship with
valid and appropriate than the person.
sons and daughters.
123. All of the following
So in this case listen to the situations allow
wife and discuss about the confidentiality and privacy of
wishes of the patient. Try to the patient to be breached
find out whether patient was except?
competent or not when he
expressed such wishes. a. When the patient
consents to allow personal
When no advance health details to be revealed to a
directive is present and no third party
spouse or family members or
career is present, a medical b. If there are other health
superintendent or authorized professionals who have a
medical officer can decide for legitimate therapeutic
life saving emergency surgery interest in the care of the
patient including medical victim of a serious offence,
students then an appropriate authority
such as the police should be
c. If there are other health informed to reduce likely
professionals who have a associated risks.
legitimate therapeutic
interest in the care of the 4. Where disclosure of the
patient excluding medical information is required or
students permitted by operation of the
law.
d. If there is overriding
public interest 5. Remember, the duty of
confidentiality includes
e. Where disclosure of the patients under the age of 18
information is required or years who do not want their
permitted by operation of parents notified.
the law
A breach of this duty made
Answer detail: lead to civil action for
The correct answer is b. damages.

There are few instances when 124. A 65-year-old male


the patient's privacy and presented with ischemic
confidentiality may be cardiac chest pain.
breached. These include: Electrocardiographic changes
were consistent with inferior
1. When the patient consents STEW. Emergency angiogram
to allow, personal details to was performed which showed
be revealed to a third party triple vessel disease. He
such as insurance companies underwent coronary artery
etc. bypass graft next day. He is
commercial driver and is
2.1f there are other health keen to know if he can
professionals who have a continue driving.
legitimate therapeutic interest
in the care of the patient (this What will be the most
does not include medical appropriate advice?
students).That is, another
doctor may read the case a. He is unfit to drive for 2
notes. weeks
b. No commercial driving
So option b is correct choice for 3 months
here. c. He can drive as private
car as long as he feels fine
3. If there is overriding public d. No driving restriction
interest. e. Write a letter to road
safety department
This is not well defined, but if
the patient was about to Answer detail:
commit, or has committed a The correct answer is b.
serious crime including
murder, rape, child abuse or After coronary artery bypass,
an act of terrorism, or was the a person should not drive a
commercial vehicle for 3 b. Azithromycin every
months and private vehicle for month
4 weeks. c. Ciprofloxacin every day
for first 6 months
This is driver's responsibility d. Penicillin injection every
to notify the road safety year
department about their e. No further antibiotics
condition and health needed
practitioners are not legally
required to do this in Answer detail:
Australia. The correct answer is a.

125. What is most common If a child develops acute


cause of hepatitis C in rheumatic fever, he/she is at
Australia? high risk of getting another
streptococcal throat infection
a. Blood transfusions and thus develop acute
b. Intravenous drug abuse rheumatic fever.
c. Needle stick injuries at
workplace The most important step to
d. Vertical transmission prevent this from happening
from mother to baby is regularly monthly injection
e. Male to male sexual of penicillin. These injections
transmission should be continued for many
years and should only be
Answer detail: stopped on a specialist doctor
The correct answer is b. advice.

Most common cause of Azithromycin and


hepatitis C in Australia is ciprofloxacin are not used for
intravenous drug abuse in the prevention of acute rheumatic
community. fever.

Intravenous drug abusers are 127. A 65-year-old male,


at the highest risk of acquiring commercial driver by
hepatitis C and other profession, presented with
infections like HIV, hepatitis B left hemiparesis and left
and syphilis by needle sharing homonyms hemianopia. He is
and using already used making good recovery.
needles. Which of the following is
most appropriate step
126. A 14-year-old child regarding driving?
developed acute rheumatic
fever after untreated a. Permanent restriction to
streptococcal throat infection drive
b. Driving assessment
Which of the following is the supervised by an
best measure to prevent occupational therapist c.
future attacks of acute Do not drive for two weeks
rheumatic fever in this child? d. Refer to neurologist to
decide for fitness to drive
a. Penicillin injection every e. Continue driving until
month police is notified
common in women and young
Answer detail: adults (below 60).
The correct answer is b.
Adenocarcinomas often occur
A person is unfit to drive in more peripheral lung
following a stroke or transient locations and may also be
ischemic attack until the associated with a history of
resolution of neurological smoking.
deficit.
Squamous cell carcinoma and
Once there is no residual small cell cancer are strongly
deficit and there is minimal associated with smoking.
risk of recurrence by
commencing secondary 129. You are a local GP in an
prophylaxis, patient should be Aboriginal Community. You
assessed by an occupational are seeing more patients with
therapist for fitness to drive. mental health issues. One of
your patients has committed
Any patient who has suicide 4 weeks after he
persistent hemianopia after visited you. You plan to
stroke, he/she must be screen patients for
assessed by an occupational depression.
therapist as well as an
ophthalmologist fitness to Which ONE of the following
drive. question has the highest
After assessment it will be specificity for depression?
decided whether this patient
fulfills the requirements to a. Do you feel sad?
continue driving a commercial b. Over the past 2 weeks,
vehicle. have you felt down,
depressed or hopeless?
128. Which of the following is c. Do you suffer from any
the most common type of mental illness?
lung cancer in people who d. Do you drink alcohol?
never smoke? e. Do you smoke?

a. Adenocarcinoma Answer detail:


b. Squamous cell The correct answer is b.
carcinoma
c. Small cell carcinoma The depression is quite
d. Non-small cell lung common in Aboriginal
cancer communities. A "yes" answer
e. Benign pulmonary to following two questions
nodule has, more than 100%
sensitivity and 12.5%
Answer detail: specificity to the diagnosis of
The correct answer is a. depression.

Adenocarcinomas are the most 1-Over the past 2 weeks, have


common type of lung cancer you felt down, depressed or
occurring in never smokers. hopeless?
Adenocarcinoma is also more
2-Over the past 2 weeks, have ELISA, also referred to as an
you felt little interest or enzyme immunoassay.
pleasure in doing things?
The most commonly used
So if you plan to do screening confirmatory test for HIV is
for depression in Aboriginal the Western blot.
Community, these two
questions have the highest 131. A 34-year-old male
sensitivity and specificity. presented for health check-
up. He is heavy smoker,
130. A 65-year-old male alcoholic and non-obese. His
presented to your office for blood pressure is 170/95.
blood tests he had last week. Serum cholesterol is
He had high-risk sexual 6.2mmol/L.
behavior in the past and anti- His father died at the age of
HIV antibodies are detected 50 with myocardial infarction.
in his serum. Which of the following carries
the highest risk of coronary
What is the most appropriate artery disease?
management?
a. Hypercholesterolemia
a. Give him antibiotics b. Hypertension
b. Start zidovudine c. Smoking
c. Repeat the test in 4 d. Alcohol
weeks e. Family history of heart
d. Enzyme immunoassay disease
and western blot
e. Give prophylaxis against Answer detail:
pneumocystis carinii The correct answer is a.

Answer detail: Hypercholesterolemia


The correct answer is d. accounts for between 30 to
40% of all coronary artery
The diagnosis of HIV infection disease related deaths.
depends on the demonstration
of antibodies to HIV and/or Hypertension accounts for
the direct detection of HIV or between 20 to 25% of all
one of its components. coronary artery disease
related deaths.
Antibodies to HIV generally
appear in the circulation 3-12 Smoking accounts for about
weeks following infection. 17% of all deaths related to
coronary artery disease.
Requesting anti-HIV
antibodies on a blood test is Risk of heart disease
done for screening purpose. If increases in patients with
the antibodies are detected, family history of heart disease
further confirmation is however it is lower than the
needed. risk of coronary artery disease
with hypercholesterolemia,
The standard blood screening hypertension and smoking.
test for HIV infection is the
Alcohol can result in breach this trust by entering
hypertension and cause into a sexual relationship with
cardiomyopathy. However it is a patient, regardless of
not the correct response in whether the patient has
this situation. consented to the relationship.

So in this patient, It may also be unethical and


hypercholesterolemia carriers unprofessional for a doctor to
the highest risk of coronary enter into a sexual
artery disease. relationship with a former
patient, an existing patient's
132. A health practitioner, who career or a close relative of an
is your colleague as well, has existing patient, if this
been involved in sexual breaches the trust the patient
relationship with one of his placed in the doctor.
24 year old female patient.
The female has consented to 133. A 28-weeks pregnant lady
the relationship. comes to a general practice,
with a presenting complaint
Which one of the following of a gush of clear fluid. On
statement is correct Speculum examination,
regarding this situation? premature rupture of
membranes is confirmed with
a. It is not permissible to closed cervix.
have sexual relationship with
a patient even if she has In addition to transferring to
consented tertiary care, what is the
most appropriate
b. It is permissible to have management?
sexual relationship with a
patient as she has consented a. Betamethasone
b. Nifedipine
c. A doctor can only have c. Salbutamol
sexual relationship with d. Cardiotocography
his/her former patients e. Urgent ultrasound

d. A doctor can only have Answer detail:


sexual relationship with The correct answer is a.
his/her exiting patients
This patient is at 28 weeks of
e. A sexual relationship gestation and presented with
with the career of the patient gush of clear fluid. Speculum
is not unethical examination has confirmed
premature rupture of
Answer detail: membrane (PROM).
The correct answer is a
Approximately, 50% of PROM
Good medical practice relies progress to labor within 24
on trust between doctors and hours and 80% within seven
patients and their families. days. The most important next
step would be to transfer this
It is always unethical and patient to tertiary care
unprofessional for a doctor to hospital as soon as possible.
Also, give corticosteroid
(Betamethasone) therapy if Influenza vaccination for
delivery prior to 34 weeks pregnant women is also
likely, for fetal lung maturity. recommended by the Royal
Australian and New Zealand
Cardiotocography (CTG) is College of Obstetricians and
usually not available in Gynaecologists, the National
general practice settings, and Health and Medical Research
it can be done while in the Council and the Australian
hospital. Induce labor if CTG is Technical Advisory Group on
abnormal or infection present. Immunizations.

There is no role of salbutamol Immunization is advised for


and nifedipine as she is not in all pregnant women during
the labor and does not require any stage of pregnancy.
tocolytics.
Vaccination early in the
134. A 25-year-old pregnant annual flu season, generally
nurse presented for routine around March, is optimal, but
antenatal checkup in her first can occur at any time. So
trimester. She is concerned there is no need to defer the
about her Influenza vaccination till third trimester
vaccination as she never got or after delivery.
this vaccination in the past.
She is not allergic to any food Pregnant females, due to
or anything else. decreased immunity, are at
increased risk of severe attack
What would be the most of influenza therefore the only
appropriate advice to this use of mask is not the most
female regarding her suitable advice.
influenza vaccination?
Influenza virus is spread via
a. Defer the vaccination till respiratory droplets so hand
third trimester washing has no role in its
b. Defer the vaccination prevention. Influenza
until after delivery vaccination is contraindicated
c. Defer the vaccine, but in people with severe egg
ask the nurse to wear mask allergy or severe allergic
regularly d. Defer the reaction to previous influenza
vaccine, but ask the nurse to vaccine dose.
wash hands frequently
e. Give vaccine now So all pregnant women should
be vaccinated against
Answer detail: influenza. The
The correct answer is e. killed/inactivated vaccine that
is usually given by
The Western Australian intramuscular route, is safe
Department of Health throughout pregnancy while
recommends that all pregnant live attenuated type, given by
women be offered influenza intranasal route should be
vaccination as part of routine, avoided in pregnancy.
comprehensive, antenatal
care.
135. A 38-year-old woman exercise therapy and
presents to your clinic for pharmacotherapy
advice. She has glucose 5-BMI more 35-40 with co-
intolerance and BMI 35. All morbidities consider
other blood tests are pharmacotherapy plus gastric
reported normal. surgery
6-BMI more than 40 gastric
What would be the best surgery
approach?
Co-morbid conditions to
a. Diet and exercise consider in the above
therapy guidelines include established
b. Gastric bypass surgery coronary heart disease,
c. Life style modification presence of other
and orlistat atherosclerotic diseases, such
d. Review in 2 years e. as peripheral arterial
Metformin diseases, abdominal aortic
aneurysm, and symptomatic
Answer detail: carotid artery diseases, type 2
The correct answer is c. diabetes, and sleep apnea.

This patient has stage 3 Bariatric surgery can be


obesity and abnormal glucose considered for patients with
tolerance test. The best severe obesity (BMI 40kg/m2)
approach would be to consider or those with moderate
life style modification (dietary obesity (BMI 35 kg/m2)
habits, physical activity, and associated with a serious
behavior modification) and medical condition.
commence orlistat.
Orlistat (Xenical) is a lipase
Glucose intolerance is usually inhibitor and a potent, slowly
managed with diet and reversible inhibitor of
exercise therapy alone. If pancreatic, gastric, lipases
patient is able to lose weight, and phospholipase A2, which
glucose intolerance will are required for the hydrolysis
improve itself. of dietary fat into fatty acids
and monoacylglycerols.
General principle of
management of obesity The drug acts in the lumen of
include: the stomach and small
intestine by forming a
1-BMI 25-26.9 diet and covalent bond with the active
exercise therapy alone site of these lipases.
2-BMI 27-29.9 without co-
morbidities diet and exercise 136. A 16-year old girl is
therapy alone brought with a deep
3-BMI 27-29.9 co-morbidities laceration on the forearm in
diet and exercise therapy if Australian Capital Territory
failed consider (ACT) hospital emergency.
pharmacotherapy You explained her the
4-BMI 30-35 with or without complications of not suturing
co-morbidities diet and it, and she seems to
understand. She still refuses
to have sutures. Her mother suture the wound under nerve
agrees with you to suture it. block.

What will you do in this The legislation regarding the


situation? age of the child to consent is
different in each state.
a. Suture the wound under
general anaesthesia In South Australia, the age of
b. Put stern taps and apply consent for medical treatment
a bandage is 16, in New South Wales, it
c. Get a psychological and is 14 and in ACT, it is 18.
mental assessment
d. Send to the family court 137. Which of the following
e. Suture the wound under would increase the risk of
nerve block breast cancer in post-
menopausal women?
Answer detail:
The correct answer is e. a. Hypertension
b. Hyperlipidemia
The age at which a person c. Obesity
becomes an adult in Australia d. Breastfeeding up to six
is 18 years. Consent for the months
medical treatment of patients e. Physical activity for 30
less than 18 years old is minutes daily
provided by parents.
Answer detail:
In general, Australian law The correct answer is c.
recognizes that individuals
aged 18 years and over have Obesity is a risk factor for the
full legal capacity, such that development of breast cancer
they are capable of making in post-menopausal women.
decisions relating to their Hyperlipidemia and
health care. Before that age, Hypertension, both are not
parents (or legal guardians) related to the increased
are entitled to consent to their likelihood of breast cancer.
child's medical and dental
treatment. Exclusive breastfeeding for up
to 6 months and active
If the situation is complicated lifestyle decreases the risk of
or unable to be resolved, then breast cancer development.
the jurisdiction for decision-
making lies with the Family 138. A 62-year-old male
Court of Australia and the presented for absolute
Supreme Court. cardiovascular risk
assessment. He has past
In this case, health medical history include type
practitioner and the mother of II diabetes mellitus and his
the young girl agree on a albumin creatinine ratio is 5
treatment, and it should be mg/mmol. He is on metformin
provided. Also, this is in the 500 mg twice daily and
best interest of the child in gliclizide 60 mg daily for
this emergency situation. So diabetes.
What will you next? partners in last 3 years. She
has been diagnosed with
a. Tell him that he is chlamydia trachomatis on this
already a high risk for presentation.
cardiovascular disease
and do not need risk What will you do next?
calculation
b. Do absolute risk a. Treat her and do contact
calculation tracing for all contacts in last
c. Start him on insulin 6 months
d. Reassure b. Treat her and do contact
e. Increase medication tracing for all contacts in last
dose 12 months
c. Treat her and her
Answer detail: current partner only
The correct answer is a. d. Tell her to give you the
names of her boyfriends
People with any of the e. Ask her to notify the
following are at high risk for health department
cardiovascular disease and do
not need absolute risk Answer detail:
calculation: The correct answer is a.

- Diabetes and age more than The current recommendations


60 years about chlamydia infection of
- A serum cholesterol more urogenital tract includes:
than 7.5 mmol/L
- Moderate to severe chronic - Treat the patient with oral
renal failure antibiotics.
- Systolic blood pressure 180 - Do contact tracing and treat
mmHg or more and diastolic patients all sexual contacts in
blood pressure 110 mmHg or last 6 months.
more. - Notify health department
- Familial (GP responsibility).
hypercholesterolemia. - Repeat testing for chlamydia
in 3-12 months for reinfection.
So this patient is at high risk
for cardiovascular disease and
need management
accordingly. There is no need
to do absolute risk
assessment.

In people with diabetes,


simultaneous treatment with
blood pressure lowering and
lipid modifying therapy is
recommended.

139. A 22-year-old woman


presented to your clinic for
assessment. She is sexually
active and has had 5 sexual

You might also like